Docsity
Docsity

Prepare for your exams
Prepare for your exams

Study with the several resources on Docsity


Earn points to download
Earn points to download

Earn points by helping other students or get them with a premium plan


Guidelines and tips
Guidelines and tips

NUR509 FNP Board Study Guide REVIEW -AANP>Chamberlain College of Nursing, Study Guides, Projects, Research of Nursing

NUR509 FNP Board Study Guide REVIEW -AANP>Chamberlain College of Nursing

Typology: Study Guides, Projects, Research

2022/2023

Available from 03/11/2023

hesigrader002
hesigrader002 🇺🇸

4.1

(38)

1.7K documents

1 / 140

Toggle sidebar

Related documents


Partial preview of the text

Download NUR509 FNP Board Study Guide REVIEW -AANP>Chamberlain College of Nursing and more Study Guides, Projects, Research Nursing in PDF only on Docsity! NUR509 FNP Board Study Guide REVIEW -AANP>Chamberlain College of Nursing PREVENTION/HEALTH PROMOTION/IMMUNIZATION Previously unvaccinated adults age 19-59 with diabetes should be vaccinated against Hepatitis B LIVE VACCINES o MMR   Patients born before 1957 have likelihood of immunity due to natural infection Two doses 1 month apart for those never immunized Varicella Zostavax Intranasal Flu Mist Avoid these with Pregnancy, immune suppression and with HIV (CD4 count < 200) – case by case situation Rotavirus o Avoid with SCID (severe combined immunodeficiency) HX of Anaphylactic reaction Immunization to avoid Neomycin IPV, MMR, varicella Streptomycin, polymyxin B, neomycin IPV, smallpox Baker’s yeast Hepatitis B Gelatin, neomycin Varicella zoster Gelatin MMR HEPATITIS B  Chronic Hep B can lead to hepatocellular carcinoma, cirrhosis and continued infectivity  Childhood Hep B vaccines began in 1982  3 dose series 0, 1, 6 months  If not vaccinated and exposed – HBIG and series  If vaccinated and exposed – single dose vaccine IMMUNIZATION PRINCIPLES  Community (herd immunity) o Immunize those who can be to protect those who cannot be immunized  Active immunity o Resistance developed in response to an antigen (either infection or vaccine)  Passive immunity o Immunity conferred by an antibody produced in another host (infant of mother or immune globulin Immunize unless sending to the hospital in an ambulance PNEUMOCOCCAL IMMUNIZATION  PCV13 associated with greater immunogenicity  PPSV23 not licensed for children under 2  Indications: chronic lung disease, chronic cardiovascular disease, diabetes, chronic liver disease, chronic alcohol abuse, smokers, malignancy, chronic renal failure, asplenia, sickle cell, immunocompromised, HIV.  PCV13 followed by PPSV23 one year later and then again at 65 o Exception: HIV (8 weeks later)  If PPSV23 before age 65, repeat in 5 years 5 A’S OF SMOKING CESSATION 1. Ask about tobacco use 2. Advise to quit 3. Assess willingness to make a quit attempt 4. Assist in quit attempt 5. Arrange follow-up SMOKING - PACK YEAR HX Number of packs-per-day (PPD) Multiplied by # of years smoked LEVEL OF PREVENTION  PRIMARY o Goal : preventing the health problem, the most cost-effective form of healthcare o Example : immunizations, counseling about safety, injury and disease prevention  SECONDARY o Goal : detecting disease in early, asymptomatic, or preclinical state to minimize its impact o Example : screening tests, such as BP check, mammography, colonoscopy, ASA in hx MI  TERTIARY o Goal : minimizing negative disease induced outcomes Example: in established disease, adjusting therapy to avoid further target organ PHARMACOLOGY RANDOM FACTS Capsaicin cream can be used to treat pain in trigeminal neuralgia and post herpetic neuralgia ASA irreversibly suppresses platelet function for up to 7 days NSAIDs Avoid in heart failure, GI bleeding, kidney disease Inhibit prostaglandin synthesis – we need prostaglandins to keep things running smoothly Long term use – document informed consent such as ↑ risk of MI, stroke, emboli, GI bleed, acute renal failure Consider PPI, H2RA, Misoprostol DRUG -DRUG INTERACTION  http://medicine.iupui.edu/clinpharm/ddis/clinical -table/  Substrates : drugs that are metabolized as substrates by the enzyme o CYP450 3A4 substrates: Sildenafil, Atorvastatin, simvastatin, venlafaxine, alprazolam o CYP450 ↓ by 30% after age 70  Inhibitors : drugs that prevent the enzyme from metabolizing the substrates (↑ drug concentration) o Macrolides (Clarithromycin, Erythromycin) – can lead to substrate induced toxicity (Ex: atorvastatin 20mg may be like 300mg) o Antifungals (ketoconazole, fluconazole) o Cisapride (propulsid – no longer in US) o Cimetidine (Tagamet) o Citalopram (Celexa) o Grapefruit (statins, erythromycin, calcium channel blocker, antivirals, amiodarone, benzodiazepines, Cisapride, carbamazepine, buspirone)  Activators: drugs that increase the enzyme's ability to metabolize the substrates (↓ drug) o St John’s wort - antiretrovirals, contraceptives and cyclosporine; Also lead to Serotonin syndrome when combined with FIRST-PASS EFFECT  Drug is swallowed and absorbed into small intestine where it enters portal PHARM FACTS  Pharmacokinetics – absorption, distribution, metabolism, elimination of a drug  Pharmacodynamics – biochemical and physiological effects of drugs on the body or disease (this does not change as a person ages) SAFETY ISSUES  PPI - ↑ risk of fractures o Prilosec interacts with warfarin  TZD – cause or exacerbate CHF  Bisphosphonates – erosive esophagitis  Statins – do not mix with grapefruit juice  Clindamycin – high risk C. Diff  Thiazide diuretics contraindication sulfa allergy o Chlorthalidone is longer acting and preferred over HCTZ  Spironolactone can lead to gynecomastia  DC ACEI/ARB if pregnant and are excreted in breast milk  Alpha blockers are only first-line in makes with HTN and BPH NARROW pnw THERAPEUTIC INDEX cs PHARMACOLOGY ANTIBIOTIC ANTIBIOTIC THERAPY LINCOSAMIDE - CLINDAMYCIN  Gram +, Aerobes, Anaerobes  Associated with C. Diff FLUOROQUINOLONES  Cipro: Gram -; atypical pathogens  Levaquin: Gram +/-; Atypical, DRSP  Achilles Tendon rupture (esp. with steroid use  Contraindicated – less than 18, pregnancy, breast feeding, myasthenia gravis  QT prolongation, hypoglycemia CEPHALOSPO RIN  1st generation: Gram +; Beta-lactam; Cephalexin, Cefadroxil Group A Strep, S. aureus – not MRSA – Keflex (pregnancy UTI, cellulitis, impetigo)  2nd generation: Gram +/- Broad spectrum – Ceftin, Cefzil – otitis media, rhinosinusitis, CAP, chronic bronchitis  3rd generation: Gram – with weak Gram +; Beta- lactam; Cefixime  Extended 3rd generation: Gram +/-; Beta- lactam; Rocephin, Cefdinir – gonorrhea, PID, pyelonephritis, otitis media  Cross reactivity between PCN and AUGMENTI N  Gram +/-, beta-lactamase. NO MRSA  High-dose 3-4g/day amoxicillin needed for drug-resistant Strep pneumoniae (DRSP)  Clavulanate as beta-lactamase inhibitor so amoxicillin can work on H. influenzae, M. catarrhalis TETRACYCLI NE  Gram -; Atypicals; MRSA  Do not use in pregnancy or children < age 9  May cause permanent discoloration of teeth and skeletal defects if used in last half of pregnancy  Treat for acne age 13-14 as all teeth have erupted o Do not use for mild comedones – start with OTC topicals such as salicylic acid and benzoyl peroxide o Try RX topicals benzamycin, Retin A and azelaic acid cream first for 2-3 months  Photosensitivity  Esophageal ulcerations (swallow with full glass of water)  Take on empty stomach  May decrease effectiveness of birth control pills MACROLIDES  Atypical pathogens  Associated with potential QT prolongation and ↑ risk of CV death  Contraindicated in myasthenia gravis  Drug-drug interactions (anticoagulants, digoxin, theophylline, select statins)  Erythromycin GI side effects are common  Macrolide allergic: doxycycline, quinolones PENICILLIN  Diarrhea, C. Diff, Vaginitis, Stevens- Johnson syndrome  Avoid amoxicillin for patients with mono (generalized rash)  Dicloxacillin – mastitis & impetigo  Anaphylaxis and angioedema are type 1 IgE  PCN allergic: macrolides GRAM NEGATIVE  H. influenzae o Cephalosporin, Augmentin, macrolides, resp. fluoroquinolones, doxycycline GRAM POSITIVE  Strep  Staph  Enterococcus PRINCIPLES OF EMPIRIC ANTIMICROBIAL THERAPY  Decision making process in which clinician chooses agent based on characteristics and site of infection  What is/are the most common likely pathogen(s) causing this infection?  What bug will this antibiotic kill?  Likelihood of resistant pathogen?  Danger if treatment failure?  What is optimal safe dose?  What duration is shortest but effective? RISK FOR ANTIBIOTIC RESISTANCE  Age <2 or >65  Antibiotic use within the last month o 3 months for pneumonia  Hospitalization within 5 days  Comorbidities  Immunocompromised HEENT - EYES TERMINOLOG Y  Palpebral conjunctiva – mucosal lining inside eyelids  Bulbar conjunctiva – mucosal lining covering eyes  Hyperopia – farsighted – distance vision intact, but near vision is blurry  Myopia – nearsighted – near vision is intact, but distance vision is blurry  Xanthelasmas – yellow plaque on the inner canthus – 50% of people have elevated lipids  Uveitis – occasionally dull painful red eye with vision changes, pupil is constricted, nonreactive and irregularly shaped. Tx with pupil HEENT MOUTH Gums may be red and swollen due to gingivitis or taking Dilantin Salivary glands – parotid, submandibular and sublingual Sialadenitis – swelling to side of face (mumps) Sialolithiasis – calculi or stone – painful lump under tongue - ↑ fluids, moist heat, NSAIDs, antibiotics if infected, surgery to remove stone Torus palatinus – painless bony protuberance midline on hard palate KOPLIK SPOTS Small-sized red papules white centers inside cheeks NOSE  Only inferior nasal turbinates are seen  Pale, boggy, bluish nasal turbinates are seen in allergic rhinitis  Sinus cavities – frontal appear by age 5 and sphenoid by age 12  Rhinitis Medicamentosa - Prolonged use (> 3 days) of topical nasal constrictor/decongestants ORAL CANCER  Ulcerated lesion with indurated margins  Relatively fixed submandibular nodes  Squamous cell cancer is most common form  Risk factors include male gender, advanced age, tobacco and alcohol abuse, HPV type 16  Screening is recommended at every dental visit DACROCYSTI TIS  Infection of lacrimal sac/tear duct usually caused by blockage  Common in infants, adults > 40  Thick eye discharge, pain, redness, swelling, warmth of lower eyelid, watery eye, excess tears  Treatment: lacrimal sac massage (downward towards mouth) 2-3 times daily, CHEILOSIS  Painful skin fissures and maceration at corner of mouth o Over salivation, poorly fitting dentures, nutritional deficiencies, lupus, autoimmune disease, irritant dermatitis, squamous cell carcinoma o Remove underlying cause o If yeast – treat with azole ointment BID o If staph – treat with mupirocin ointment BID o When infection clear, use barrier cream at night  Zinc  Petroleum jelly LYMPH NODES  Cancerous nodes are non-tender  Tender nodes indicate infection RED EYE COMPLAINTS  Send to ophthalmology o Change in vision o FB sensation with inability to keep eye open o Photophobic  Keep FB sensation, scratchy, gritty o Pink eye Acoustic nerve or CN VIII Vestibulocochlear – Auditory Presbycusis – age related hearing loss; difficulty appreciating content in noisy environment HEENT Conductive Sensorineural Location Outer or middle ear Inner ear Cause Sound is being blocked (earwax, foreign objective, damaged eardrum, serous otitis media, bone abnormality – cholesteatoma) Inner ear or nerve becomes damaged (advanced age, ototoxic medications, immune disorders, trauma) Weber Result – tuning fork on forehead Sound lateralizes to affected ear (buzzing sound heard louder in affected ear) – otitis media, ceruminosis, perforation of TM Sound lateralizes to unaffected ear (buzzing sound not at all in affected ear- numbness) – Presbycusis Meniere’s disease Rinne Result – mastoid then front of ear Negative (BC > AC) Normal (AC > BC) or positive **can hear longer in front of ear than on mastoid bone Treatment Often self-resolves post cerumen impaction removal, post URI or AOM resolution. Rarely, further pharmacologic or surgical intervention is needed **Temporary Hearing aids or cochlear implants possible options with expert communication **Permanent BACTERIA OF THE EAR  S. pneumoniae – usually causes most significant symptoms and is least likely to resolve without antibiotics o Resistance to low dose amoxicillin, certain cephalosporins and macrolides o Mechanism of resistance is alteration of intracellular protein-binding sites  Use high dose amoxicillin and select cephalosporins o Risk factor is recent antibiotic use  H. influenza and M. catarrhalis o Gram negative capable of producing beta lactam ring o High rate of spontaneous resolution o H. influenza is most common found in mucoid and serous middle ear infection  Treatment o No recent antibiotics  Amoxicillin high dose TID  Augmentin BID  Cefdinir o Recent antibiotic use  Augmentin BID  Levofloxacin or Moxifloxacin EAR  Bones – Malleus, incus and stapes  Tympanogram – test for presence of fluid inside middle ear (straight line vs peaked shape)  Pinna injuries – refer to plastics  Tragus – small cartilage flap of tissue in front of ear  Ceruminosis – carbamide peroxide HEENT EMERGENCIES CONTINUED Name Cause Signs/Symptoms Diagnostics Treatment s Concerns Battle Sign Trauma Raccoon eyes (periorbital ecchymosis) and bruising behind the ear (mastoid area) that appears 2-3 days after trauma. Search for clear golden serous discharge from ear or nose. Rule out basilar or temporal bone skull fracture. Basilar skull fractures can cause intracranial hemorrhage. Refer to ED Clear Golden Fluid Discharge from Nose/Ear Basilar skull fracture Clear golden fluid discharge from nose/ear Test fluid with urine dipstick – positive for glucose (plain mucous or mucopurulent drainage is negative) Refer to ED Peritonsill ar Abscess Severe sore throat with difficulty swallowing, odynophagia (pain on swallowing) and a “hot potato voice” Unilateral swelling of peritonsillar area and soft palate. Affected area is markedly swollen and appears as a bulging red mass with uvula displaced away from mass. Accompanied by malaise, fever and chills. Refer to ED Diphtheri a Sore throat, fever and markedly swollen neck (bull neck) Low grade fever, hoarseness and dysphagia. Posterior pharynx, tonsils, uvula and soft palate are coated with gray to yellow colored pseudomembrane that is hard to displace. Very contagious. Contact prophylaxis required. Refer to ED Epiglottiti s Sore throat, fever, muffled voice, drooling, stridor, hoarseness “hot potato”, “thumb sign” Refer to ED Hib vaccine eradicated in kids HEENT Name Cause Signs/Symptoms Diagnostics Treatments Concerns Corneal Abrasion Acute onset severe eye pain with tearing. Reports feeling of foreign body sensation on surface of eye. Always ask about contacts Fluorescein dye. Linear or round Flush eye with sterile normal saline. Evert eyelid to look for foreign body. Topical antibiotic trimethoprim- polymyxin B (Polytrim), Ciprofloxacin (Ciloxan), Ofloxacin (Ocuflox) to affected eye 3-5 days. Do not patch eye. Contact Lens-Related Keratitis – acute onset red eye, blurred vision, watery eyes, photophobia, foreign body sensation Hordeolum (Stye) Abscess of hair follicle and sebaceous gland in upper or lower lid. Internal – inflammation of meibomian gland Acute onset swollen, red and warm abscess on the upper or lower eyelid. May spontaneously rupture and drain purulent exudate. Hot compresses x 5-10 minutes BID – TID until drained May need dicloxacillin or erythromycin PO QID Can cause cellulitis of eyelid Chalazion Chronic inflammation of meibomian gland Gradual onset of small superficial nodule on upper eyelid that feels like a bead and is discrete and moveable. Painless. Can slowly enlarge over time. If large enough, can press on cornea and cause blurred vision. I&D, surgical removal or intrachalazion corticosteroid injections. Pinguecula Chronic sun exposure Raised yellow to white small growth in the bulbar conjunctiva (skin covering eyeball) next to the cornea If inflamed, refer to ophthalmologist for RX of weak steroid eye drops only during exacerbations. Use artificial tears as needed for irritation. Good quality sunglasses 100% UVA and UVB. Remove surgically if encroaches on cornea and affects vision Pterygium Chronic sun exposure Yellow triangle wedge-shaped thickening of conjunctiva that extends across the cornea on the nasal side. Surfer’s eye. Can be red or inflamed at times. May complain of foreign body sensation on the eye. Non-cancerous Subconjunctiv al hemorrhage Coughing, sneezing, heavy lifting, vomiting, local trauma, spontaneously Blood trapped underneath the conjunctiva and sclera secondary to broken arterioles. Watchful waiting and reassurance of patient Primary Open- Angle glaucoma Elevated intraocular pressure Risk factors: African ancestry, Type 2 DM, advanced age, family hx of POAG Gradual onset of increased IOP greater than 22 due to blockage of drainage of aqueous humor inside eye; most common in elderly; usually asymptomatic during early stages. Gradual change in peripheral vision then central vision (tunnel vision). May complain of missing portions of words when reading. If cupping of optic disc– IOP is too high Refer to ophthalmologist. Check IOP with tonometer. Normal 8 to 21 (>30 is very high) Betimol 0.5% - ↓ aqueous production (beta/alpha blockers) Latanoprost – topical prostaglandin SE of med: bronchospasm, fatigue, depression, heart failure, bradycardia Contraindicated asthma, COPD, heart failure Most common glaucoma. Blindness due to ischemic damage to retina. 2nd leading cause of blindness Risk factors: postural hypotension, hx of fungal conjunctivitis, white race Macular gradual damage to pigment of macula Asymptomatic in the early stages. Complains of gradual or sudden and Refer to ophthalmologist. Pt is given Amsler grid to check vision loss daily Most common cause of vision loss. Leading HEENT degenerati on (area of central vision) results in severe visual loss to blindness painless loss of central vision in one or both eyes. Straight lines become distorted or curved (scotoma). More common in smokers to weekly Atrophic (dry form) or exudative (wet form) – wet is responsible for 80% cause of blindness in elderly Risk factors: age, smoking Sjogren’s Syndrome Chronic autoimmune disorder Decreased function of lacrimal and salivary glands. Persistent daily dry eyes and mouth (xerostomia) for > 3 months. Eyes have sandy or gritty sensation. Uses OTC artificial tears TID OTC tear substitute TID Refer to ophthalmology, dentist and rheumatology HEENT Name Cause Signs/Symptoms Diagnostics Treatments Concerns Otitis Externa Pseudomona s aeruginosa S. aureus Proteus spp. Enterobacteriaceae External ear pain, swelling, discharge, pruritus and hearing loss. Hx of recent activities that involve swimming or getting the ear wet. Ear pain with manipulation of external ear or tragus. Purulent green discharge. Erythematous ear canal Physical exam, if malignant consider CT, radionucleotide bone scan, gallium scan, MRI Polymyxin B-neomycin-hydrocortisone suspension (Corticosporin Otic) 4 gtts QID x 7 days; Ofloxacin otic or Ciprofloxacin otic ear drops BID x 7 days. Keep water out of ear. (No steroids if eardrum ruptured) Preventative: Domeboro (boric) or alcohol and vinegar Mononucleosis EBV (incubation is 30-50 days) Malaise and fatigue, Fever, pharyngitis, lymphadenopathy. Hx of sore throat, enlarged posterior cervical nodes and fatigue for several weeks. May have abdominal pain. Virus is shed through saliva 50% have splenomegaly (normal spleen is 1 x 3 x 5, weighs 7 oz and lies between 9 and 11 ribs) CBC: atypical lymphocytes and lymphocytosis; LFT: abnormal for several weeks; Heterophile antibody test: Monospot +; Large cervical nodes; Erythematous pharynx with inflamed, sometimes white exudate; Hepatomegaly and splenomegaly; red maculopapular rash Limit physical activity for 4 weeks. Abd US is splenomegaly/hepatomegaly. Repeat US in 4 to 6 weeks. Avoid amoxicillin. Avoid close contact, kissing, sharing utensils. If airway obstruction, hospitalize with high dose steroids Peak ages 15-24 3F’s and an L Fever, Fatigue, Pharyngitis and Lymphadenopat hy Aphthou s stomatiti s unknown Painful shallow ulcers on soft tissue of mouth that usually heal within 7-10 days Aphthous ulcer – canker sore Magic mouthwash – liquid diphenhydramine, viscous lidocaine and glucocorticosteroid Geographic tongue Benign physiologic al variant Multiple fissures and irregular smooth areas looking like a topographic map. May have soreness after eating or drinking acidic or hot foods Leukoplakia Poor fitting dentures, chewing tobacco, alcohol abuse Slow growing white plaque with firm to hard surface, slightly raised on tongue, floor of mouth or inside cheek Precancerous lesion refer Meniere’s disease Increased pressure within endolymphatic system Present with episodes of vertigo with a sensation that the room is whirling about – preceded by decreased hearing, tinnitus and feelings of increased pressure. Characterized by repeat attacks that last minutes to hours and can be related to food and drinks, mental and physical stress and variations in menstrual cycle Largely diagnosis of exclusion. Horizontal nystagmus usually towards the affected ear with rapid correction to midline. Weber lateralizes to unaffected ear. Rinne’s AC>BC; performing pneumatic otoscopy in affected ear can elicit symptoms. Romberg + Fukuda marching step test is positive with drift Antihistamines such as meclizine, antiemetics or benzodiazepines can minimize symptoms, thiazide diuretics can decrease pressure load in ear and prevent but not treat attacks. Corticosteroids have also been demonstrated to be helpful Prevention: avoid ototoxic drugs, protecting ears from loud noise and limiting sodium intake Risk factors include: use of ototoxic drugs such as aminoglycosides, long- term high dose salicylate use, certain cancer drugs and long-time exposure to loud noise. Can also have Meniere’s symptoms with certain situations but not the disease HEENT towards affected ear. Dix-Hall Pike is occasionally also positive indicating BPPV SKIN/INTEGUMENTARY SYSTEM SMALLPO X  “Eliminated” 1977  Infection targets respiratory and oropharyngeal surfaces. Incubation period of 2 weeks.  Flu-like signs and symptoms with large nodules in center of face, arms and legs. Symptomatic treatment  Mortality rate 20-50% PRESSURE ULCER  Stage I: nonblanchable erythema on intact skin  Stage 2: presence of epidermal or dermal skin loss; can appear as intact blister  Stage 3: full-thickness skin loss with exposure of some amount of fat, ulcer has crater-like appearance  Stage 4: full-thickness skin and tissue loss; would exposes muscle, bone and tendons SKIN CANCER ASSESSMENT A – Asymmetry B – Border irregularity C – Color (brown, black, red, white, blue) D – Diameter >6mm (pencil eraser) E – Evolving/Elevated (most are new)  > 2 features ABCDE 100% sensitive, 98% specific  Melanoma: dark colored moles with uneven textures – may be pruritic  Acral lentiginous melanoma: most common in AA/Asian. Nailbeds, palms/feet  Subungual hematoma: direct trauma to nailbed causing bleeding between bed and finger – trephination – SKIN LESIONS  Annular – in a ring (Bull’s eye lesion – Lyme disease) – central clearing  Bulla – blisters > 1cm w/fluid (burn)  Clustered - lesion occurring in a group without pattern (herpes)  Confluent or Coalescent- multiple lesions blending together (psoriasis vulgaris)  Cyst – raised, encapsulated fluid filled lesion (intradermal lesion)  Lichenification – skin thickening usually found over pruritic or friction areas  Linear - lesion distribution in streaks (poison ivy)  Macule – flat non-palpable are of discoloration <1cm (freckle)  Maculopapular – both color and small papules or raised skin lesions ranging from erythematous to bright pink  Nodule – solid lesion > 0.5 – 2 cm (> 2 cm tumor)  Papule – solid elevation <0.5 cm (mole) o Smooth papule dome shaped with central umbilication with white plug (molluscum contagiosum)  Patch – flat, nonpalpable area of skin discoloration larger than macule (vitiligo)  Petechiae – < 1cm (thrombocytopenia)  Plaque – elevated, variable shape >1cm (psoriasis)  Purpura – flat, red-purple discoloration that does not blanch with pressure  Pustule – vesicle like lesion with purulent content (impetigo)  Reticular – netlike cluster  Scale – raised superficial lesions that flake with ease (dandruff)  Scattered – generalized over body without specific pattern (viral exanthem – rubella/roseola)  Vesicle – clear fluid (herpes)  Wheal – circumscribed area of skin edema (urticaria) DERMATOLOGY ASSESSMENT TIP  Assess the entire patient, not simply the skin problem. Consider whether there is transmission/contagion risk.  Where did it start? (face, torso, extremities, genitals)  How long have you had it?  Does it itch?  Is the patient otherwise well? o Disease limited to skin such as rosacea, keratosis pilaris, seborrheic dermatitis.  Is the patient miserable, but not systemically ill? o Itch, burning, pain – scabies or shingles  Is the patient systemically ill with constitutional symptoms (fever, fatigue, loss of appetite, unintended weight loss, malaise)? o Varicella, transepidermal necrosis, Lyme disease, systemic lupus erythematosus  Primary lesions only? Primary and Secondary? o Where is oldest lesion - when did it occur? o Where is newest lesion – when did it occur? o Primary – result from disease process, has not been altered by outside manipulation or tx  Ex: vesicle – fluid filled < 1 cm (varicella, shingles, herpes) o Secondary – lesion altered by tx or progression of disease SKIN/INTEGUMENTARY SYSTEM Name Cause Signs/Symptoms Diagnostics Treatments Concerns Rocky Mountain Spotted Fever Dog/Wood Tick bite Rickettsia rickettsii high fever, chills, severe headache, n/v, photophobia, myalgia, conjunctival injection, arthralgia; 2-5 days after onset - rash (petechiae) starts on hands/feet to trunk (palmar rash) Antibody titers to rickettsia Punch biopsy CBC, LFT, CSF Doxycycline 100mg BID for 7-14 days – can be fatal if not started on treatment within 8 days. Remove tick by grasping closest to skin and apply steady upward pressure. Can be fatal (3- 9%) Highest in southeastern/sout h central regions of US Most common Apr - Sept Brown recluse spider bite Fever, chills, nausea and vomiting. Red, white and blue sign; central blistering with surrounding gray to purple discoloration at bite surrounded by blanched skin surrounded by large areas of redness Ice at time of bite. Local debridement, antibiotic ointment, elevation, loose immobilization. NSAIDs Prevention: look before putting body part into places where spiders hide such as footwear and boxes Found in midwestern and southeastern US Erythema Migrans (Lyme disease) Borrelia burgdorfer i (tick) Expanded red rash with central clearing (bulls- eye), feels hot to touch and rough texture; common areas belt line, axilla, popliteal, groin FLU-LIKE symptoms Rash appears 7-14 days after deer tick bite Spontaneously resolves Enzyme immunoassay (EIA) Indirect immunofluorescence assay (IFA) Early: Doxycycline BID x 14-21d (maybe 28 days) (Alt: Amoxicillin or Ceftin) Avoid doxycycline in children due to teeth staining. Stage 1: single painless annular lesion Stage 2: AV heart block, Bell’s palsy Stage 3: joint pain 1 yr. after infection NE regions of US Systemic infection with organ shutdown Guillain- Barre Migratory arthritis Meningococcemi a Neisseria Meningitides Gm – diplococci Spread via respiratory sudden onset of sore throat, cough, fever, headache, stiff neck, photophobia, and changes in LOC abrupt onset of petechial to hemorrhagic rashes Lumbar puncture: CSF Blood/throat cultures CT or MRI of brain Rocephin 2G IV q12h + Vancomycin IV q12h Hospital isolation and supportive tx Close contact prophylaxis: Rifampin BID x2d and meningococcal vaccine Death within 48 hours Medical emergency – REFER College students in dorms ↑ risk asplenia, sickle cell, HIV Varicella / Zoster Herpes-zoster (Chickenpox or Shingles) – spread by direct contact or inhalation Chickenpox: Fever, pharyngitis, malaise Pruritic vesicular lesions beginning at head expanding to trunk (vesicles and crust) – most common on thorax Shingles: lesions at various stages along dermatome Contagious 1-2 days before rash until crusted Viral culture PCR for ZDV Vaccine: >60 Tzanck smear confirms shingles Varicella: Acyclovir within 24-48 hrs. of eruption (avoid ASA and NSAIDs) Acyclovir or Valacyclovir x 10d for initial outbreak and then x 7 days for flare-ups Post herpetic neuralgia: TCA, anticonvulsant, gabapentin, lidocaine or capsaicin cream to intact skin Herpes zoster ophthalmicus (corneal blindness) CN V - photophobia, eye pain, blurred vision Maligna nt Melanom a A – Asymmetry B – Border irregularity C – Color (brown, black, red, white, blue) D – Diameter >6mm (pencil eraser) E – Evolving/Elevated (most are new) May be pruritic Needs biopsy by derm. Refer to Dermatology Acral Lentiginous Melanoma – African American and Asians, located on nail beds, palmar and plantar surfaces Risk factors, family hx of melanoma, sun exposure, tanning beds, lots of nevus, light skin/eyes Basal Cell Carcinom a Most common skin cancer Waxy, pearly domed nodule, usually distinct borders with or without telangiectasia; white, light pink, brown or flesh colored. Papule, nodule with or without central erosion PUT ON (Pearly papule, Ulcerating, Telangiectasia, On the face, scalp, pinnae, Nodules, slow growing Risk factors: light- colored skin, Australian decent, sun- exposed area Metastatic risk low, significant tissue destruction risk without treatment. Slow growing – common in fair skin types Excisional biopsy SKIN/INTEGUMENTARY SYSTEM Actinic Keratosis Slow-growing; Scaling, dry, round, flesh- colored lesions on skin that do not heal; usually sun- exposed areas; sizes range from microscopic to several centimeters Clinical diagnosis fluorouracil cream (5FU cream); 5% imiquimod cream, topical diclofenac gel Liquid nitrogen, laser resurfacing or chemical peel Gold standard: refer to derm for biopsy Pre-cancerous Precursor to squamous cell carcinoma Erythema Multiforme (Stevens- Johnson syndrome) Rare reaction to medications (NSAIDs, Sulfa, antiepileptic), infection (herpes or Mycoplasma pneumoniae) and malignancies Lesions like “bulls-eye”, erupt suddenly Hives, blisters, petechiae, purpura, hemorrhagic lesions and sloughing of epidermis. Could have prodrome of fever with flulike symptoms 1- 3 days before rash appears (palmar rash) Toxic Epidermal Necrolysis (TEN) (>30% of skin) allopurinol, anticonvulsants, sulfa, NSAIDs SKIN/INTEGUMENTARY SYSTEM Name Cause Signs/Symptoms Diagnostics Treatments Concerns Psoriasis Inherited; excessive mitotic grown of epithelial cells Erythematous papules and plaques “fine- silvery scales” found over elbows, scalp, knees, gluteal folds; pitting of fingernails Auspitz sign: Pinpoint areas of bleeding remain in the skin when a plaque is removed. Chronic Koebner phenomenon – scar formation in places not near typical psoriasis First Line: Medium potency topical corticosteroid, Second Line: Vitamin D derivatives Topical retinoids (tazarotene), Tar preparations; UVB light and topical tar may induce remissions **BB can exacerbate Psoriatic arthritis: painful red, warm and swollen joints. Guttate psoriasis: severe form resulting from Gp A strep infection Tinea Versicolor Yeast - Pityrosporum orbiculare or Pityrosporum ovale. Hypopigmented round macules on chest/shoulders/back; appear after skin is tanned from sun asymptomatic KOH slide: hyphae & spores “spaghetti & meatballs” Topical selenium sulfide Ketoconazole (Nizoral) BID x 2w Oral antifungals Atopic Dermatitis (Eczema) Inherited pruritic rash Pruritic rash on hands, flexural folds, and neck; well-demarcated round-to-oval erythematous coin-shaped plaques exacerbated by stress and environment; Starts as small vesicles that rupture leaving red, weeping lesions that become lichenified and itchy Clinical diagnosis Skin lubricants/hydrating baths to alleviate dryness Topical steroids: Mild – hydrocortisone 1- 2.5% Medium – triamcinolone Med/High potency (Halog) x 10d Oral antihistamines for pruritis Formation of fissures and risk of infection Triad – allergies, eczema, asthma Acute cellulitis Strep pyogenes Staph Aureus (MSSA or MRSA) Infection of dermis and subcutaneous fat with heat, redness and discomfort in the region, poorly demarcated Non-purulent: Cephalexin 500mg QID, dicloxacillin QID or Clindamycin x 10 day MRSA: Bactrim, Doxycycline or Clindamycin Td booster if >5yrs **Good follow-up; 48 hrs. after initial tx. Refer if s/s don’t resolve, cellulitis not responding to tx, spreading quickly, DM, immunocompromised Osteomyelitis, sepsis Cutaneous abscess, furuncle, carbuncle Staph Aureus (MSSA or MRSA) Skin infection involving hair follicle and surrounding tissue (heat, redness and discomfort) Carbuncle – multiple abscesses C&S I & D and warm soaks Bactrim, Doxycycline or clindamycin MSSA: Dicloxacillin or Cephalexin Bactroban for folliculitis Erysipelas S. pyogenes Sudden onset one hot, indurated, erythema with clear demarcation. Usually on lower legs or cheeks Non-purulent: Keflex or dicloxacillin Hospitalization for infants, immunocompromised Bite wounds Dogs & Cats (P. multicoda) gram negative Humans (Eikenella corrodens) Human bite dirtiest of all Cats higher risk of infection than dogs Rabies: skunks, raccoons, foxes, coyotes: Immune globulin and vaccine Quarantine domestic animals for up to 10d Wound C&S Augmentin 875mg x 10d (PCN allergic: doxycycline or Bactrim + Flagyl or clindamycin) Clean, no sutures, tetanus Follow-up 24- 48 hours Watch for closed- fist injury (infection of the joints) 80% cat bites become infected Hidradenit is Suppurati ve Bacterial infection of axillary sebaceous gland Staph aureus Acute onset painful, large, red nodules and papules under one or both axilla that become abscessed C & S of drainage Mild: Chlorhexidine; Clindamycin 1% for 12 weeks; Tetracycline 500 BID; doxycycline or minocycline BID for 7- 10 days Recurrences and scars Impetigo Staph Aureus Strep pyogenes Nonbullous – erythematous macule evolves into pustule ruptures leaving honey crusted exudate. Bullous – clear, yellow fluid ruptures within 1-3 days leaving a rim of red with moist base C & S of crusts/wounds Nonbullous - mupirocin ointment 2% x10d (treats select gram + organisms) Cephalexin or dicloxacillin QID x 10d Azithromycin if PCN allergic 250 x 5d or No school until 48-72 hours after treatment initiation working with animals or bioterrorism – symptoms are flu- like with cough, chest pain with cough, hemoptysis, dyspnea, hypoxia, shock CARDIOVASCULAR SYSTEM MURMURS  Turbulent blood flow through great vessels or across a heart valve  Timing of murmur – systole or diastole  Location of murmur – aortic or mitral  Systolic murmurs – MR Peyton Manning AS MVP o M itral Regurgitation- “pan systolic”/ holosystolic. Heard at apex, radiates to axilla, loud or high-pitched blowing (use diaphragm) o P hysiologic Murmur-  Hemic like in severe anemia, dehydration  Becomes louder when lying down o A ortic Stenosis- second ICS to right of sternum, radiates to neck, harsh/noisy murmur (use diaphragm) – avoid physical exertion due to ↑ risk sudden cardiac death; monitored by echo and surgical valve replacement; LVH; congenital defect in kids; acquired could be from prior rheumatic fever; angina, syncope, heart failure o M itral Valve Prolapse- mid to late systolic murmur with mid systolic click  Diastolic murmurs – ARMS o A ortic Regurgitation- high-pitched, second ICS to right of sternum blowing (use diaphragm) o M itral Stenosis- low pitched, apex of heard or apical area; also called “opening snap” “crescendo” (use bell)  All diastolic murmurs – abnormal  APETM (valve locations) o Aortic – 2nd ICS right upper border sternum o Mitral – apex or apical area of heart, PMI, 5th left IC space  If radiates to axilla the mitral valve is closest to axilla  If radiates to neck the aortic valve is closest to neck  If a valve fails to open it is stenotic  When a valve fails to close it is incompetent HEART SOUNDS  Motivated Apples o Motivated – systole – S1 closure of AV - mitral/tricuspid valve (lub)  M (mitral valve)  T (tricuspid valve)  AV (atrioventricular valves) o Apples – diastole - S2 closure of semilunar - aortic/pulmonic valves (dub)  A (aortic)  P (pulmonic)  S (semilunar)  S3 - Pathognomic for CHF; possible normal for adolescent athletes and pregnancy; possible thyrotoxicosis; always abnormal if occurs >35yo “Kentucky” – best heard at pulmonic area (AKA ventricular gallop or S3 gallop)  S4 – LVH; normal finding in some elderly; occurs late is diastole and best heard at apex with bell (Tennessee) (atrial gallop or atrial kick); poorly controlled HTN; unstable angina  Split S2 is best heard at pulmonic area – healthy athlete STETHOSCOPE  Bell – low tones (S3, S4), mitral stenosis  Diaphragm – mid to high pitch tones, lung sounds, mitral regurgitation, aortic stenosis GRADING OF MURMURS  GRADE I/VI – barely audible  GRADE II/VI – audible but faint  GRADE III/VI – moderately loud, easily heard (as loud as S1)  GRADE IV/VI – loud, associated with a thrill  GRADE V/VI – very loud, heard one corner of the stethoscope off the wall  GRADE VI/VI – loudest, heard without a stethoscope CARDIOVASCULAR SYSTEM ADDITIONAL PEARLES  BP = HR x SV x PVR  Left side of heart is higher level arterial system – right side is lower pressure thus abnormalities in the cardiac exam are more likely to arise from left-sided heart problems  Deoxygenated: Superior vena cava → right atrium → tricuspid valve → right ventricle → pulmonary valve → pulmonary arteries → lungs (RBCs pick up oxygen and release carbon dioxide in alveoli) Oxygenated: pulmonary vein → left atrium → mitral valve → left ventricle → aortic valve → body  Left ventricle is most likely cardiac chamber to hypertrophy o Left ventricular hypertrophy is common form of HTN TOD o PMI shift (downward and lateral) common in LVH (normally heard at 5th ICS MCL)  2nd chamber most likely to hypertrophy is left atrium  Common pathologic murmurs to arise due to aging – aortic stenosis  Most common regurgitate murmur – mitral regurgitation (mitral valve incompetent – means it doesn’t close properly) which causes decreased cardiac output o Symptoms of low cardiac output  Dyspnea with exertion  Chest pain  Orthopnea (virtually never respiratory, almost always heart failure – could be LVH and mitral regurgitation)  Syncope and near-syncope (when cardiac, generally caused by aortic stenosis or hypertrophic obstructive cardiomyopathy)  Idiopathic hypertrophic subaortic stenosis – type of cardiomyopathy – autosomal-dominant pattern o Cardiac exam in mitral regurgitation (MR) includes holosystolic murmur with blowing quality typically GR II-III/IV with predictable pattern of radiation to the axilla  Holosystolic – takes up all of systole and is the same intensity throughout systole  Sudden unexpected finding on exam- how did I miss this in health history? o Onset o Location/radiation o Duration o Character o Aggravating factors o Relieving factors o Timing o Severity FIBRINOLYSIS CONTRAINDICATION:  Absolute : prior intracranial hemorrhage, cerebral vascular lesion, neoplasm, ischemic stroke in last 3 months, aortic dissection, active bleeding, intracranial or intraspinal surgery within 2 months, severe uncontrolled HTN  Relative : chronic uncontrolled HTN, significant HTN on presentation, ischemic stroke < 3 months, traumatic or prolonged CPR, major surgery within 3 weeks, recent internal bleeding within 2-4 weeks, non-compressible vascular punctures, pregnancy, active peptic ulcer disease, oral anticoagulant therapy CARDIOVASCULAR SYSTEM ASCVD – high intensity LDL > 190 – high intensity Diabetes – moderate intensity 10 yr. ASCVD risk > 7.5% - moderate to high intensity **Not enough data of benefit > 75 Take at night with baby ASA CARDIOVASCULAR SYSTEM Name Cause Signs/Symptoms Diagnostics Treatments Concerns Acute Coronary Syndrome: (STEMI, NSTEMI and unstable angina) Unstable angina: vasoconstricti on, nonocclusive thrombus, inflammation or infection Midsternal chest pain, squeezing, tightness, crushing, heavy pressure, band- like, numbness/tingling left jaw/arm, diaphoresis w/ cool/clammy skin. Pain is provoked by eating heavy meal or exercise. Continues to have pain or discomfort at rest; Women present with fatigue, sleep disturbance, dyspnea, anxiety, weakness, back pain, nausea, syncope EKG Beta-blockers, ACEI, aldosterone antagonists STEMI – transmural MI with subsequent Q waves. NSTEMI – subtotal occlusion Stable angina – pain is predictable. Congestive Heart Failure (Left sided) MI, CAD, HTN, fluid retention, valvular abnormalities, arrhythmias Lungs have crackles bibasilar and S3 heart sound Crackles, cough, dyspnea, dullness to percussion, paroxysmal nocturnal dyspnea, orthopnea, non- productive cough and wheezing (“left = lung”) Chest x-ray, (Kerley B lines) EKG, CPK, troponin, BNP, CMP, echo Monitor weight daily, Avoid ETOH, stop smoking diuretics, ACEI or ARBs, beta- blockers if HFrEF, aldosterone antagonist Limit sodium intake (2-3 grams) Fluid restriction 1.5-2 L daily NYHA classify degree of physical disability Class I – no limitations Class II – activity results in fatigue, exertional dyspnea Class III – limitation in physical activity Class IV – symptoms at rest EF < 40% systolic failure (HFrEF) EF > 40% diastolic failure (HFpEF) Meds that contribute to heart failure: amlodipine, metoprolol (but they need it), actos/Avandia (glitazone), NSAIDs Congestive Heart Failure (right sided) MI, CAD, HTN, fluid retention, valvular abnormalities, arrhythmias JVD (normal < 4cm), enlarged spleen, enlarged liver causing anorexia, nausea, and abdominal pain, lower extremity edema (“right = GI”) Bacterial Endocardit is Gm + Viridans streptococcus, staph aureus Fever, chills, and malaise associated with new murmur and abrupt onset CHF Subungual hemorrhages, petechiae on palate, painful nodes on fingers or feet (Osler nodes), nontender red spots on palm/soles (Janeway lesions); fundoscopic exam Roth spots or retinal hemorrhages; hematuria REFER to cardiologist Blood cultures x 3 CBC, Sed rate >20 mm/h Antibiotic prophylaxis is no longer recommended for MVP, GU or GI Recommended for: Previous hx of endocarditis – dental procedures, prosthetic valves – resp. procedures, congenital heart disease Amoxicillin 2 grams PO Adult 1 hr. prior Amoxicillin 50mg/kg 1 hr. prior PCN allergic: Clinda 600mg, Biaxin 500mg, Keflex 2 grams,? macrolide Valvular destruction, myocardial abscess, emboli Dissecting Abdominal Aortic Aneurysm Pulsating-type sensation in abdomen or lower back pain. Sudden onset severe chest/back pain increasingly sharp and excruciating. Distended abdomen with hypotension Abdominal ultrasound Incidentally: CXR may show widened mediastinum, tracheal deviation, obliteration of aortic knob Surgical. If less < 4cm monitor yearly with CT. Risk factors: male > 60, smoker, uncontrolled HTN, white race, genetic disease such as Marfan syndrome Cardiac Arrhythmi as Atrial Fib Risk factors: HTN, CAD, ACS, caffeine, nicotine, hyperthyroidism, alcohol intake, heart failure, LVH, PE, COPD, sleep apnea May be asymptomatic; May be more than 110 bpm on palpation (if hemodynamically unstable – chest pain, hypotension, heart failure, cold clammy skin, acute kidney failure with new A Fib call 911 12-lead EKG TSH, electrolytes, renal function, 24h Holter monitor, echo Avoid stimulants Search for underlying cause; Refer to cardiologist. CHA2DS2- VASC score > 2 needs anticoagulants CHF, HTN, Age > 75, Diabetes, Stroke/TIA, Vascular disease, Age 65-74, Sex (female); possible CCB, BB/digoxin to regulate HR Warfarin – A fib INR 2-3; Valves 2.5- 3.5 Most common, classified as SVT – lead to stroke Paroxysmal AF: episodes terminate < 7 days INR >4-5 hold 1 dose and/or reduce maintenance CARDIOVASCULAR SYSTEM dose Paroxysmal Supraventricular Tachycardia (PSVT) Digitalis toxicity, alcohol, Hyperthyroidism, caffeine, illegal drugs Abrupt onset of palpitations, rapid pulse, lightheadedness, shortness of breath and anxiety HR range from 150-250bpm EKG Vagal maneuvers, carotid massage, ice water to face. If WPW or symptomatic call 911 May be seen in Wolf Parkinson White Syndrome CARDIOVASCULAR Name Cause Signs/Symptoms Diagnostics Treatments Concerns First Degree a ? Teer a SHE fi i i Second degree heart block, type I cwenckepacn/mMopitz D SS SS A ee Se it : ri + Progressive protengation oF rm intervat Sarees See compre Second degree heart bIOcK. type 2 cmopitz 2> . bo srocee ons + + Wp ereemes ons ; ee ey | ee Third degree heart block fetes mpren trrosson sve oma, i ‘ iL SPs CARDIOVASCULAR Mitral Valve Prolapse Systolic murmur Fatigue, palpitations, chest pain, lightheadedness aggravated by heavy exertion; May be asymptomatic; associated with pectus excavatum, hypermobility of joints, arm span greater then height (Marfan’s syndrome) S2 “click” – mid to late systolic murmur Cardiac echocardiogram with doppler flow study Asymptomatic needs no treatment MVP w/ palpitations tx with BB, avoid caffeine, alcohol and cigarettes. Holter monitoring can be used to determine arrhythmias ↑ risk thromboemboli, TIA, AF and ruptured chordae tendineae Hyperlipidemia Risk factors: HTN, premature heart disease (women < 65 and men < 55) DM, dyslipidemia, low HDL, cigarette smoking, obesity, microalbuminuria, CAD, PAD Total cholesterol: Normal: less than 200 Borderline: 200-239 High: greater than 240 HDL: greater than 40 (low HDL is generally from ↑ carb and ↓fat diet) LDL: less than 100 Triglycerides: less than 150 – pancreatitis associated with > 1000 (if triglycerides are > 500 treat triglycerides first – fenofibrate, niacin, lovanza) (avoid alcohol and Tylenol; could be metabolic syndrome, DM, familial, alcohol abuse, hyperthyroidism, kidney disease, medications Fasting lipids starting at age 20 (and every 5 yrs.) >40 screen every 2-3 yrs. If HLD: screen at least annually Exercise, lose weight, eat healthy fats, eliminate trans fats, decrease junk food, stop smoking, DASH diet Target is to lower LDL first unless ↑ triglycerides (>500) – STATINS Triglycerides are raised by alcohol and sugar Simvastatin interactions: grapefruit, fibrates, antifungals, macrolides, amiodarone Monitor for myalgias Consider stopping cholesterol at age 80 if symptoms, but statins are excellent at keeping atherosclerosis stable so can prevent MI or CVA Statins can cause memory loss, confusion. Rhabdomyolysis Acute breakdown of skeletal muscle, acute renal failure Triad of muscle pain, weakness and dark urine Muscle pain and aches persistent without associated muscular exertion CK, urinalysis (myoglobinuria/proteinur ia), BUN, creatinine, potassium, EKG Risk factors: obesity, Nonalcoholic Fatty Liver Disease (NAFLD) (Fatty Liver) Triglyceride fat deposits in liver Usually asymptomatic; may have hepatomegaly If symptomatic, fatigue and malaise with RUQ pain. Associated with obesity, metabolic syndrome, DM and HLD Annual labs show increase of ALT and AST, negative hepatitis A, B, C. Weight loss, diet Discontinue alcohol, avoid hepatotoxic drugs (acetaminophen, statins) REFER GI for Liver biopsy (gold standard) ***can progress to cirrhosis diabetes, metabolic syndrome, HTN, certain drugs Most common liver disease in US - #1 reason for liver transplants Varicose Veins Inherited venous defect, leg crossing, wearing constrictive garments, prolonged standing, heavy lifting Tortuous dilated superficial veins; leg aching, mild edema at the end of the day and in warm weather most often great saphenous vein affected Laser venous ablation, sclerotherapy, surgery Women are affected 2x as much as men HEART BLOCK  First Degree AV Block : prolonged PR>0.2 seconds o If the R is far from P – FIRST DEGREE  Second Degree Type I (Wenckebach) : PR is progressively longer until it drops o Longer, Longer, Longer, drop then you have WENCKEBACH  Second Degree Type II : PR constant but drops QRS periodically o If a QRS don’t get through, then you have MOBITZ II  Third Degree : complete, no pattern between PR and QRS o If Ps and Qs don’t agree, then you have THIRD DEGREE CARDIOVASCULAR PULMONARY SYSTEM “uawjean dn days 0} peau ey) pue joUOD eyenbapeu! eyeo!pul AW UONED!D9Uy endsed Jo sn Sulseeiou! Jo Juanbely “Awenes WO\dWAS UO Spuadap AljsUalU| JUEWIZEL, “SWOIGLUAS JOJ Papasu Se (JoJ@INGIe “3'9) IS|UDSe-elaq Supoe-LOUS :NOLWOIGAIW 3NIsay euuyse ofB9]e yf Adexoyjzounwu eplsuog joods eUyyse UM 3INSUOD ‘ei 9 YANaIy p sdayg sae jy + plosays poyeyul @soOp-WNIpa| pjorays B10 SUA 7-0 ADV + Jayooig 4990/4 ausi}04Ne| aualnoyna) 40 1SIUOSE-219Q 4aya0|q Plosays payeyul 40 1s1U03e-e19q Bunoe Zuo 101319 onenes ‘SS0D-WNIDW ‘su92-Buo] sayNE = 40 |SIUOBe-e}9q 40 194004 + ploweis pajeuul Sunoe-Buo| auaijoxne} Plolays pajeyut OS0p-USIH reyyie 4o \slu082 @sop-UsIH : rena + ej0q Bune Bug (anop ul SUA 0 39V Ploisys p9yeyut + poses PI|eyUl \souse|p STEN a GEHTS SUA 7-0 39V @SOp-WNIpE|l See 1edsa) OES a to bse 5 a Jap|suod tere p t=) SUA +0 39 SUA TT-S 39V FURIES 4940019, SUA 7-0 IDV seinen wea) Pjosays jes0 auaLoyna| seen set ety + + ploiais pareyul Paneer Jax001q ‘osop-y Sik +z se ous 4ay9010 z yeu a] auayjoyn| seaneuleyy ia dusuoYNo} uKIowo19 + plosays pajeuul . SHA TT-S 39 PY PICAeIS PSIeuuy, we (eM eytertite -at=it)) eS0p U asiuoge ‘asop-M07 euau04ne7 Se rier] roanewauly nea eal ronpeuony reanewany Eimear + Pr re asiuoze. ‘@SOp-USIH 4820/4 ploseys peyeyul plos9ys payey By9q BuNoe-Suo] POMojor @uaHN}04N9B| ‘@SOp-UNIDEW SSOp-M0T (Cerin) + plosays payeyul se “ + plow4s pa|eyul 40 ysjuoze Pond Sar EI eSOp-U3iH SUA TT-S JV asop-unipa, Freq BunzeBuo} i SPOMajodd feagewoyy + plosays payeuul sa9v TIV - . - 980P-MO7 y qsiuoBe_ DIJO ez SUA TES 39¥ sarBiaye J leq Sunoe-Buo) : “ t dls gra? qewlznjewo 1 plosoys poyeyuy SUA +ZT 3DV 8258 Jepisuog @SOp-UN IPE; essa saiBialye 1 —pue— spanajald Bekg qewiznjewo ysluoBe € d3ls ! gee Jepisuoa e19q BUNDe-BUO| SUA +ZT 3DV ! gkET —pue— + plosais payeyur I gad Plosays @SOp-UBIH a255 le10 + ysluoze seo 7 v d3ls ! agge ey2q Sul02-Bu0} SUA +2T 3OV 1 age + plouays payeuut Sao asop-4glH I Bag g das 1 Lu SHA +2T 39V 1 322 258 aEo see 9 d3als Bujusem xoq y9eI/q & eney sygyT Sujupezuoo squese uojeurquioo pue syEVT IV 1 928 3 "SYUOW 9 0} & AJ9A9 [01}U09 SSaSSe 94} 9]G2}S AIDA J] I gS z OE eee CEM Le Cees tg I Fe I 8 8 CM a Ames MCE lke I fe eouas9ype Jood ajqissod ssasppe ySnoye pazeoipul se dn dazg I Ly UO!}EDIPSIAI Ayieq -BWYSY U9}SISIOdg SICH ITTOSE SUIT LSID /S10"djaHewysy}2 :a1ou! pue suEld UOROE EUNyySe ‘sapin3 UOReoIpaUy sousseJ91 YIND VINHLSV SNISVNVIAI OL HOVOUddV ASIMdALS Tap PULMONARY SYSTEM Name Cause Signs/Symptoms Diagnostics Treatments Concerns Pulmonary Emboli Hx of A fib, estrogen therapy, smoking, surgery, cancer, pregnancy, long bone fractures and prolonged inactivity Sudden onset dyspnea and coughing. Cough may be productive of pink-tinged frothy sputum. Tachycardia, pallor and feelings of impending doom Impending Respiratory Failure - Asthmatic Exacerbation Tachypnea, tachycardia or bradycardia, cyanosis and anxiety. Patient appears exhausted, fatigued, diaphoretic and uses accessory muscles to help with breathing. Cyanosis and quiet lungs. May speak in 1- 2-word sentences Epi stat. 911 oxygen, albuterol nebs, parenteral steroids, antihistamines and H2 blocker COPD – can include chronic bronchitis and emphysema Alveolar damage from loss of elastic recoil of lungs, exposure of irritants Airflow limitation Chronic cough (2 years), chronic sputum production, shortness of breath worse with physical exertion, progressive symptoms, barrel chest, weight loss; hyperresonance upon percussion, tactile fremitus and egophony is decreased; CXR may show hyperinflation; bullae sometimes present; coarse crackles Alpha-1 antitrypsin deficiency screening < 45yrs ** pack year smoker – COPD** Spirometry FEV1/FVC <0.70 post- bronchodilation. Classification of severity determined by FEV1 CXR only when trying to R/O pneumonia Smoking cessation, pneumonia & flu vaccine, Acute exacerbation: SABA, LABA, ICS Acute prednisone 40mg/day x 5-10 days **↑ dyspnea, ↑ sputum volume & purulence may need antibiotic; higher risk for Strep pneumonia & H. influenzae –Augmentin (avoid due to GI upset) Cefdinir, macrolide (avoid due to CV risk); resp. fluoroquinolone (↑ tendon rupture) High risk: > 2 exacerbations in last year, FEV1 < 50%, hospitalized for COPD in past year **rarely see under age 40 Chronic bronchitis: cough with excessive mucous for 3+ months for 2+ years Complex patients with co-morbidities: Multiple meds, drug- drug interactions, drug- disease interactions Community- Acquired Pneumonia (CAP) Strep pneumoniae; H. influenzae; Mycoplasma; Chlamydophil a pneumonia; Cystic fibrosis: Pseudomas aeruginosa (Gm-) Sudden onset high fever w/ chills, productive cough and purulent sputum (rust-colored if strep pneumo). c/o pleuritic chest pain w/ coughing and dyspnea Rhonchi, crackles or wheezing with dullness over affected lobe, ↑ tactile fremitus and egophony; abnormal whispered pectoriloquy Chest x-ray-lobar consolidation (note: middle lobe is anterior chest by nipple) CBC: leukocytosis (>10.5) look for anemia S. pneumoniae: macrolides, doxycycline, DRSP: high dose amoxicillin, resp. fluoroquinolones. Minimum 5 days (most 5-7 days) No comorbidities: Macrolides, doxycycline Comorbidities: resp. fluoroquinolones, or macrolide plus beta-lactam Flu vaccine > 50yo Pneumo vaccine >65yo Smokers – H. Influenza S. pneumoniae affects young and old Lung cancer can present as recurrent pneumonia Atypical Pneumoni a Mycoplasma; Chlamydophil a pneumonia; Legionella Fatigue w/ paroxysmal coughing that’s nonproductive; gradual onset starting like a cold; most continue to work/school regardless of symptoms Wheezing with diffuse crackles/rhonchi, rhinorrhea, erythematous throat Physical, chest x-ray shows diffuse infiltrates, CBC Macrolides, resp. fluoroquinolones, doxycycline; antitussives, fluids/rest ***Legionella – contaminated by inhaling mist from a water source. Also accompanied by GI Acute bronchitis Virus causing inflammation of Upper respiratory Sudden onset new cough that’s dry and possible small amts sputum; frequent paroxysms of coughing, possible low- grade fever, wheezing and chest pain w/ History, possible chest x-ray Symptomatic: dextromethorphan, Tessalon PERLES, guaifenesin, severe wheezing Atrovent or Exacerbation of asthma Pneumonia from secondary infection PULMONARY SYSTEM tract cough; cough keeping him awake at night albuterol inhaler consider steroids 40mg for 3-5 days ***On rare occasion macrolide or doxycycline Pertussis “whooping cough” Bordetella pertussis bacteria (Gm -) Cough lasting longer than 14d with 1 of following: paroxysmal coughing, inspiratory whooping w/o cause. Can last months. 3 stages: catarrhal, paroxysmal, convalescent. Most infectious early in disease Nasal swab for culture and PCR Pertussis antibodies by ELISA CBC: lymphocytosis (80% lymphocytes in WBC) 1st line: macrolides Chemoprophylaxis for close contacts Respiratory precautions Antitussives, mucolytics, rest and hydration Tdap booster > 11yo Complications: Sinusitis, OM, pneumonia, fainting, rib fractures ENDOCRINE SYSTEM ENDOCRINE FACTS Endocrine works as negative feedback – low level of active hormone stimulates production. Hypothalamus stimulates anterior pituitary gland into producing stimulating hormones (FSH, LH, TSH) – these hormones tell organs to produce hormones. Hypothalamus: coordinates nervous and endocrine system by sending signals, produces neurohormones that stimulate or stop production Pituitary Gland: Posterior pituitary: secretes antidiuretic hormone and oxytocin which are made by hypothalamus and stored and secreted by posterior pituitary Thyroid gland: uses iodine to produce T3 (huge impact on metabolism – 5x) and T4 (small changes affect TSH) Parathyroid glands: produce PTH which is responsible for calcium balance of body by regulating calcium loss or gain from bones, kidneys and GI tract Pineal gland: pea-sized gland in brain that produces melatonin. ADDITIONAL DIABETES CONSIDERATIONS  A – Aspirin (81-162 mg daily) in most, esp. men > 50 and women > 60 with DM and > 1 additional CVD risk factor such as HTN, smoking, family hx  B – BP controlled > 2 agents including ACEI/ARB and thiazide  C – Cholesterol: statin therapy > 40 or hx ACS. Goal Lipids < 100 Creatinine (renal function): serum creatinine, calculated GFR and urine microalbumin annually  D – Diet: limit trans and saturated fat. Schedule with dietician if needed Dental care: reinforce  E – Exercise: > 150 min/week (walking) + resistance exercise 3x week Eye exam: annually (diabetic retinopathy) o Neovascularization (new growth of fragile arterioles in retina), microaneurysms, cotton wool spots, soft/hard exudates  F – Foot exam: visually every visit & monofilament at minimum annually  G – Goals of care Hypothalamus TRH, GnRH, CRH, GHRH, Somatostatin “On or Off” Switch Released by Anterior Pituitary TSH, FSH, LH, GH, ACTH, MSH, Prolactin, Vasopressin, and Oxytocin Target Organs Thyroid (TSH): T3 and T4 (thyroxine) – free or bound (no impact on metabolism) Ovaries/Testes (FSH/LH): estrogen, progesterone, androgens, testosterone Adrenal Cortex (ACTH): glucocorticoids, mineralocorticoids Body (GH): somatic growth Uterus (oxytocin): uterine contractions, bonding Kidneys (vasopressin): blood volume Pineal (melatonin): circadian rhythm Breast (prolactin): milk production DAWN PHENOMENON  Elevation in FBG daily early in morning from increase in insulin resistance between 4 and 8am caused by spike in GH and glucagon RANDOM OBESITY PEARLS  Orlistat – take within an hour of each meal that contains fat  Belviq – avoid with medications that have serotonergic effect  Phentermine – teratogenic effects  Serotonin is responsible for the sensation of satiety  1 pound of fat contains 3500 calories  10,000 steps is = 4-5 miles  Weight loss meds: if not achieved 5% weight loss by week 12, discontinue therapy  Weight loss of 10%+ yields immediate reduction in death from cardiovascular and cerebrovascular disease  Bariatric surgery- most dramatic weight loss is seen in first few months, calcium absorption will be reduced, rapid weight loss can contribute to gallstones, lifelong vitamin B12 supplementation is recommended SOMOGYI EFFECT  Severe nocturnal hypoglycemia stimulating glucagon to be released from the liver  High FBG by 7:00a, usually due to overtreatment with evening or bedtime insulin – more common in Type 1  Diagnosed by checking glucose at 3am for 1-2 weeks  Tx: Snack before bedtime or eliminate/lower THYROID THYROID NODULE Solitary Thyroid Nodule Palpable thyroid mass > 1 cm in diameter 5% chance of malignancy Malignant Thyroid Nodule Hx head or neck irradiation Size > 4 cm Firmness, Nontender on palpation Relatively fixed position (non-mobile) Persistent Nontender cervical lymphadenopathy Dysphonia Hemoptysis TSH, Thyroid ultrasound ↑TSH – metabolically inactive (most common) Fine needle aspiration biopsy (refer) Most cost effective ↓TSH – metabolically active Nuclear med thyroid scan Hot – metabolically active radioactive ablation or surgery Not hot – metabolically inactive Fine needle aspiration Cold usually cyst HYPERPARATHYROIDISIM Common cause in asymptomatic patient with hypercalcemia ↑ Calcium and PTH ↓ phosphorus ↓ potassium TOXIC ADENOMA Benign, metabolically active thyroid nodule Autonomously functioning adenoma: Painless thyroid nodule with undetectable TSH ENDOCRINE SYSTEM HYPOTHYROID HYPERTHYROID SKIN Thick, dry Smooth, silky REFLEXES “hung up” patellar reflex, slow arc out, slower arc back, overall hyporeflexia Hyperreflexia MENTATION “can’t make sense, thoughts too slow” “Can’t make sense, mind racing” WEIGHT CHANGE Small gain 5-10 lbs. Loss ~ 10 lbs. STOOL PATTERN Constipation Frequent, low volume, loose MENSTRUAL ISSUE Menorrhagia Oligomenorrhea HEAT/COLD INTOLERANCE Easily chilled Heat intolerance OTHER Hypertriglyceridemia Proximal muscle weakness Tachycardia, HTN THYROID  Thyroid-stimulating hormone (TSH) NL 0.4-4.0 (goal 1.2) o Evaluates hypothalamic-pituitary function  Anterior pituitary ability to detect circulating free thyroxine o When TSH normal, thyroid disease ruled out  Free T4 (free thyroxine) o Unbound, metabolically active portion of thyroxine o F/U test to confirm, support dx of hypo or hyperthyroidism with abnormal TSH  Thyroid peroxidase antibody (TPO Ab) o Test to help detect autoimmune thyroid disease o Measures antibody against peroxidase  Total T4 (total thyroxine) o Reflects the total of the protein-bound and free thyroxine o (useless – altered with medications, clinical conditions) Untreated hypothyroidism, inadequate thyroxine dose  ↓ Free T4 = ↑ TSH Untreated hyperthyroidism  ↑ Free T4 = ↓ TSH  Drugs that affect thyroid: Lithium, amiodarone, high doses of iodine, interferon-alfa, dopamine (lithium can damage thyroid)  Natural thyroid contains fixed doses of T3 and T4 and has different pharmakinetics than levothyroxine  Excessive use of levothyroxine includes bone thinning  Periodic routine screening is recommended with Down Syndrome  Hypothyroidism - ↑ LDL, hyponatremia, ↑ MCV, ↑ CK BETA BLOCKERS  Beta-adrenergic antagonist with beta 1 blockade o 1 heart  Beta-adrenergic antagonist with beta 1 blockade o 2 lungs, 2 arms, 2 legs (tremors) ENDOCRINE Name Cause Signs/Symptoms Diagnostics Treatments Concerns Thyroid Cancer ↑ risk with radiation therapy from childhood cancer (Wilms tumor, lymphoma, neuroblastoma) or low- iodine diet; family hx of thyroid cancer Single thyroid nodule usually in upper half of one lobe, may be accompanied by enlarged cervical nodes. May complain of hoarseness and dysphagia. Occurs in women 3:1 Age 20- 55 Pheochromocyto ma Random episodes of headache (mild to severe) diaphoresis, tachycardia, HTN. Episodes resolve spontaneously. Pt’s vitals are normal in between attacks. Hyperprolactine mia Sign of pituitary adenoma. Slow onset. Women may present with amenorrhea. Galactorrhea in both males and females. When tumor is large enough, pt. will complain of headaches and vision changes. Serum Prolactin ↑ Check prolactin if galactorrhea or gynecomastia – big boobs or lactating = prolactin Hyperthyroidis m (thyrotoxicosi s) Graves’ disease most common cause Tachycardia, rapid weight loss, irritability, anxious, hyperactivity, insomnia, possible HTN, a fib or PAC, sweaty, exophthalmos, diarrhea, amenorrhea, heat intolerance, fine tremors, brisk deep tendon reflexes, CHF. goiter ↓ TSH, ↑ T3/T4 If Graves: + thyrotropin receptor antibodies (TRAb). Thyroid peroxidase antibody (TPO) is positive in Graves and Hashimotos Thyroid Ultrasound Refer to Endo for RAIU Start on beta-blocker (propranolol, nadolol) to counteract tachycardia and tremors Propylthiouracil (PTU) and Methimazole (Tapazole) shrinks gland/↓ hormone production (rash, anemia, thrombocytopenia, hepatic necrosis) radioactive iodine (ablation – then need levothyroxine) Supplement with Calcium and Vitamin D 1200mg plus weight- bearing exercise ↑ incidence in women 7:1 ↑ risk for RA, pernicious anemia, osteoporosis Thyroid storm ↓ LOC, fever, abdominal pain Hypothyroidism Hashimoto’s (autoimmune), postpartum, and thyroid ablation with radioactive iodine Fatigue, weight gain, cold intolerance, constipation, menstrual abnormalities, alopecia on outer 3rd of eyebrows, may have ↑ cholesterol. May have atrial fib. Myxedema may have poor thinking/memory, hypotension, hypothermia Symptoms are very variable ↑ TSH, ↓ free T4 (normal or low T3) ** subclinical hypothyroidism has elevated TSH and normal free T4 & T3 Use ideal body weight if pt. obese.  Adults 1.6mcg/kg/day  Elderly (Start 25mcg)  Child 4mcg/kg/day  ↑ by 33% in pregnancy Check TSH about 8 weeks after tx Levothyroxine should be taken with water on an empty stomach. Avoid taking within 2 hours of calcium, iron, aluminum, magnesium. Report palpitations, nervousness, tremors Addison’s Disease Primary: ↓ cortisol and sometimes aldosterone produced by adrenal glands Secondary: Symptoms vary and develop over months. Chronic diarrhea, N/V, loss of appetite, paleness or darkening of the skin with a possible patchy appearance, muscle fatigue, weakness, slow or sluggish movement, hypoglycemia, low BP, fainting and salt craving. During crisis, AM Cortisol level K+, Na+, ACTH Abdominal CT for adrenal glands, MRI pituitary gland Corticosteroid replacement therapy Ample sodium during heavy exercise, hot climates and GI upsets. During crisis, an immediate injection of hydrocortisone is needed along with support for low BP Glucocorticoids (Cortisol) Mineralocorticoid (Aldosterone) ENDOCRINE SYSTEM Metformin unless contraindicated A1C > 9: Consider dual therapy initially A1C > 10-12: injectable insulin until less glucose toxic BG > 300: injectable insulin until less glucose toxic ***Meglitinides for irregular eating schedule DC sulfonylureas, glitazone after initiating insulin (SU plus insulin is less efficacious with more weight gain) HEMORRH OIDS Grade I – no prolapse, Grade II – prolapse upon defecation but reduce spontaneously, Grade III – hemorrhoids prolapse upon defecation and must be reduced manually, Grade IV – hemorrhoids are prolapsed and cannot be reduced manually. ↑ risk excessive alcohol, chronic diarrhea or constipation, obesity, high fat, low fiber diet, prolonged sitting, sedentary lifestyle, anal intercourse and loss of pelvic floor muscle tone. GASTROINTESTINAL SYSTEM ANAL FISSURE  Ulcer or tear of anus, most often posteriorly  Severe anal pain (razor blades) with bowel movements lasting hours after the BM – pain leads to constipation and drops of blood when wiping  Risk factors include constipation, diarrhea, childbirth, anal sex  Primary treatment is to prevent constipation - ↑ dietary fiber, laxative (mineral oil – avoid long term use due to inability to absorb A, D, E, K vitamins), sitz bath, cool compresses. If these measures fail NTG, Botox, surgical sphincterotomy CONSTIPATI ON  Idiopathic and functional  Lifestyle factors o Immobility o Low-fiber diet o Dehydration o Milk intake o Ignoring the urge to have BM  Drugs contributing to constipation o Iron supplements o Beta-blockers o Calcium channel blockers o Antihistamines o Anticholinergics o Antipsychotics o Opiates o Calcium-containing antacids  Treatment o Bowel retraining o Dietary changes o Ingest bulk forming fiber (25-35 g/day) o Increase physical activity o Increase fluid intake (8-10 glasses) o Consider laxatives HEPATITIS SEROLOGY  IgG – produce after the infection is Gone o Antibodies present (immune)  No virus / not infected  IgM – antibody you make the Minute you get infected o Acute infection – contagious  No immunity  Hepatitis A o Fecal-contaminated food/water o Anti-HAV IgG – positive  Immune to HAV o Anti-HAV IgM – positive  Acute infection o Anti-HAV IgM and IgG negative – no immunity, needs immunization  Hepatitis B o HBsAg – (surface antigen) screening for Hep B  Positive – has virus on board o Anti-HBs (surface antibody)– positive  Antibodies present / immune o Anti-HBc (totally Hep B core antibody) ***rotten to the core*** o IGM anti-HBc  Hepatitis C o Injection drug use o Anti-HCV – screening for Hep C (? Exposure – if negative order HCV RNA)  If positive – order HCV RNA or PCR to r/o chronic infection  If positive RNA/PCR has Hep. C – refer o Screen those born from 1945-1965  Hepatitis D o Requires presence of Hep. B. Can be acute/chronic o Infection of B/D increases risk for cirrhosis and liver damage anal disease. pattern on endoscopy – skin lesions azathioprine. More common in Jews Ulcerative Colitis Inflammatory disease affecting colon/rectum (Inflammation affects mucosa) bloody diarrhea with mucus. Severe “squeezing” cramping pain located on left side of abdomen with bloating and gas exacerbated by food. Relapses characterized by fever, anorexia, weight loss and fatigue. Accompanied by arthralgias and arthritis that affects large joints, sacrum and ankylosing spondylitis. May have IDA or anemia of chronic disease ↑ CRP, ESR, Leukocytosis Anemia WBCs in stool Oral aminosalicylates sulfasalazine and mesalamine (better tolerated) Corticosteroids. No antibiotics due to risk of C. Diff. Immune modulators ↑ risk of toxic megacolon and colon cancer GASTROINTESTINAL SYSTEM Name Cause Signs/Symptoms Diagnostics Treatments Concerns Zollinger- Ellison Syndrome Gastrinoma on pancreas or stomach Gastrin secretion stimulating high acid production of stomach. Develops ulcers in stomach and duodenum. Epigastric to mid- abdominal pain; stools may be tarry colored Fasting gastrin levels GERD Gastric contents regurgitate from stomach to esophagus due to reduction in LES tone, irritation of esophageal mucosa, ↑ gastric secretion Chronic heartburn over time with large/fatty meals worsening when supine. Recurrent dry cough, chronic pharyngitis, and hoarseness. Self-tx w/ OTC antacids and H2 blockers. May be due to chronic NSAID use, aspirin, or alcohol ***Anyone with GERD x 10 yrs. should be referred to GI to R/O Barrett’s HPI – Clinical Dx Endoscopy when dysphagia, odynophagia, unintended weight loss, hematemesis, melena, chest pain or choking First line treatment is lifestyle changes (avoid eating 3-4 hours before bedtime, dietary changes and weight loss) avoid triggers (mints, chocolate, alcohol, ASA, NSAIDs, caffeine, carbonated beverages) Stop smoking. Evaluate meds: BB, CCB, alpha agonist, estrogen, progesterone. 2nd line: Can combine lifestyle changes with antacids or H2 blockers (prn up to 12 hours), if no relief: PPI once a day, before the 1st meal of day (long term therapy associated with hip fx, pneumonia, C Diff.; wean off due to rebound) – If no relief after 4-8 weeks, refer to GI Chronic GERD may result in Barrett’s esophagus ↑ risk of squamous cell cancer – (followed by GI – lifetime PPI with endoscopic biopsy annually – 6 months) ***CCB – blocks calcium and ↓ electoral conductivity – smooth muscle relaxer which ↓ LES and increases GERD Irritable Bowel Syndrome (IBS) Disorder of colon (spastic colon) Intermittent episodes of moderate to severe cramping in lower abdomen, especially LLQ. Bloating with flatulence. Relief after defecation. Stools range from diarrhea to constipation. Exacerbations/remissions. Commonly exacerbated by stress. Abdominal exam tenderness in lower quadrants R/O bacterial infections ROME III criteria: recurrent discomfort 3 days/month in last 3 months with 2+ Discomfort relieved by defecation, change in stool form or appearance Increase fiber, avoid gas producing foods: beans, onions, cabbage, high- fructose corn syrup. Antispasmodics. If constipation based – trial fiber supplements, Miralax. If diarrhea- based take Imodium before regularly scheduled meals. Decrease life stress More commonly affects females Erosive Gastritis (Gastric Ulcer) H. pylori; Too much stress, alcohol or NSAIDs Intermittent epigastric pain, burning/gnawing pain. Pain worse with eating, tender at epigastrium Worrisome symptoms: dysphagia, early satiety and weight loss H. pylori negative: Stop NSAIDs, (if needs NSAIDs add PPI or misopristol). Stop alcohol. Stop smoking. Stress management. Lifestyle changes with H2 blockers the step up to PPI. H. pylori positive: triple therapy Clarithromycin + Amoxicillin + omeprazole (Flagyl if allergic to Amoxicillin) x 14 d Sudden abruption of medication can cause rebound worsening symptoms Duodenal Ulcer H. pylori Episodic epigastric pain, burning/gnawing pain. Pain relieved by food or antacids with recurrent 2-3 hours after meals. Awakens at 1-2am with symptoms H. pylori stool antigen test or urea breath test (no serological H. pylori because will test positive if ever infected) More common H. pylori is transmitted oral/fecal and oral/oral A: fatigue, nausea, anorexia, malaise, abdominal pain, dark colored urine, clay stools and joint pain for several days. Skin and sclera have a yellow tinge. Tenderness over liver with percussion Reportable to public health department Hep A: Asymptomatic; Hep A symptoms start about 28 days after exposure; Post- exposure and not vaccinated (age 1-40, give HAV) if >40 give IG within 2 weeks of exposure; avoid oral contraceptives to avoid cholestasis; avoid alcohol. Avoid working in food-related jobs for 1 week after Viral Hepatitis Fecal/oral B: sexual; mother to child; blood transfusion C: IV drug abuse, blood transfusion and deep palpation. ↑ ALT/AST up to 10x normal. Remove and treat cause. Avoid hepatotoxic agents such as Tylenol, alcohol and statins. Treatment is supportive Periodic monitoring for alpha- fetoprotein to look for hepatoma (hepatocellular carcinoma) May have aversion to smoking onset of infection Hep B: acute, self-limiting or chronic infection. Tx: first-line agents pegylated interferon alfa (PEG- IFN-a), entecavir (ETV), and tenofovir disoproxil fumarate (TDF) – Offer vaccine to those born before 1986 - cannot get vaccine if anaphylaxis to baker’s yeast. Post-exposure and vaccinated give HBV, if in the process of vaccination series, give HBIG and complete series. Unvaccinated should receive HBIG and start vaccine series within 24 hours of exposure if possible Hep C: approximately 75-85% of people who will become infected will develop chronic infection. Most common cause of liver cancer and liver transplantation. Screen adults from 1945-1965 Tx: administer antivirals such as ledipasvir-sofosbuvir (Harvoni), ribavirin, and pegylated interferon alpha 2a/2b MINI COG: Three-word recognition Clock drawing (normal or abnormal) Three-word recall Dementia if score 0-2. No dementia if >3 Exam – physical including fundoscopic and neuro exam BP, pulse Palpate head, neck, shoulders, spine Bruits If you are 35+ and develop a new headache – you bought an expensive test If ordering imaging notes should reflect: Red flag headache Change in pattern, frequency or severity of HA Worsening of HA despite therapy Unexplained neuro symptoms Headache always on the same side Onset of HA with exertion, cough, intercourse New onset > 50 yrs. HA associated with fever, stiff neck, papilledema, cognitive impairment or personality change Where does it hurt? Characteristics? Patient appearance (lights out, fetal position vs reading iPad) Duration Associated symptomso o History o o o HEADACHE EVALUATION: NERVOUS SYSTEM HEADACHES PRIMARY SECONDARY Not associated with any other diseases Associated with or caused by other conditions Migraine, tension-type, cluster **more common Tumor, intracranial bleeding, ↑ICP, meds like NTG, meningitis, giant cell arteritis Red Flag Headaches: Systemic symptoms: - Fever, unintended weight loss - Secondary HA Risk factors: HIV, malignancy, pregnancy, anticoagulation, HTN Neurological signs, symptoms: - Newly acquired symptoms, confusion, impaired alertness or consciousness, nuchal rigidity, HTN, papilledema, CN dysfunction o Acceptable abnormalities include Photophobia & phonophobia - Unequal pupil size Onset: sudden, abrupt or split-second “thunderclap” headache (subarachnoid) onset with exertion, sex, cough ↑ICP Older >50 or < 5 years Previous headache history - Less worrisome if have had HA before - “worst headache of my life” – r/o subarachnoid Rule-Out: - Subarachnoid or acute subdural - Leaking aneurysm - Bacterial meningitis - Increased ICP - Brain abscess or tumor HEADACHE DIFFERENTIAL: - Nasal stuffiness: sinusitis - Jaw claudication, fever, visual loss, pain in temple: temporal arteritis*** - Visual field defect: Optic pathway lesion (pituitary tumor) - Blurred vision on bending head: intracranial lesion - Headache with N/V: tumor - Unilateral vision loss: optic neuritis - Sweating, tachycardia: pheochromocytoma - Transient visual changes: MINI-MENTAL STATE EXAM (MMSE): - Orientation - Short term memory - Recite 3 unrelated words - Attention and calculation Spell “world” backwards or subtract 7 starting at 100 - Recall – ask to repeat the words - Write sentence - Copy design - While speaking, look for aphasia (impairment in BRAIN DAMAGE:  Apraxia : difficulty performing purposeful movements  Broca’s aphasia : “nonfluent aphasia” Pt comprehends speech and can read, but has difficulty with motor aspect of speech – word salad  Wernicke’s aphasia : “fluent aphasia” Pt has difficulty with comprehension but has no problem with speaking. Reading and writing can be impaired.  Frontal lobe damage : (intelligence, personality) dementia, memory loss, difficulty to learn NERVOUS SYSTEM Name Cause Signs/Symptoms Diagnostics Treatments Concerns Acute bacterial meningitis Strep pneumonia Neisseria meningitides Haemophilus influenza High fever, severe headache, stiff neck and meningmus. Rapid changes in mental status. Purple-colored petechial rash w/ N/V and photophobia. Worsening symptoms to lethargy, confusion and coma Lumbar puncture: CSF large WBC, ↑ protein, ↓ glucose CT/MRI CBC, CMP, Coags, Blood culture x 2 Gram stain and C&S of CSF Report to Health Department. Infants: Ampicillin or 3rd gen. cephalosporin Adults: 3rd gen. cephalosporin + Chloramphenicol >50: amoxicillin + 3rd gen. cephalosporin Prophylaxis of close contacts with rifampin or ceftriaxone Fatal if not treated Temporal arteritis (Giant cell arteritis) Autoimmune vasculitis of temporal artery Acute onset headache located on one temple, usually in older adult; induration, redness, cordlike temporal artery accompanied w/ scalp tenderness. Abrupt visual changes and/or transient blindness (amaurosis fugax). Some people may also complain of jaw pain. Elevated ESR Elevated CRP Refer to ophthalmologist or ED Biopsy is definitive test High-dose steroids 40-60mg daily (Add PPI to prevent ulcer and possibly bisphosphonate for bone health) Mean age dx 72 Permanent blindness ***women more likely than men Pt’s with polymyalgia rheumatica are at very high risk 30% Stroke (CVA) Embolic or hemorrhag ic Embolic: Acute onset stuttering/speech changes, one-sided facial weakness, hemiparesis Hemorrhagic: May have poorly controlled HTN and severe headache, N/V, and nuchal rigidity 911 – Assess ABCs Risk factors – A fib and HTN; aneurysm, anticoagulants, stimulants, sickle cell, diabetes, oral contraception, smoking. Blacks, Hispanics and Indians have ↑ prevalence Chronic subdural hematoma (SDH) bleed between dura and subarachnoid membrane History of head trauma and HA with gradual cognitive impairment (apathy, somnolence, confusion) More common in elderly & those on anticoagulation or ASA therapy Subarachnoid hemorrhage (SAH) Head trauma Sudden onset HA “worst HA ever” w/ photophobia, N/V, meningeal irritation (+Brudzinski and Kernig signs), decline in LOC. Elderly – fall Younger – MVC Sentinel headache – can occur a few up to 20 days before event. Migraine (with or w/o aura) Gradual onset throbbing headache behind one eye gradually worsening over several hours; photophobia and phonophobia. May last 2-3 days and become bilateral if not treated Aura can be paresthesia, seeing halos, metallic taste, hyperosmia, Scotomas (blind spots in visual field) etc. Positive family history and being female increases risk factors. In children, migraines can present as abdominal pain. ***Note motion sickness and migraines Migraine without aura  HA last 4-72 hrs.;  has 2 characteristics (unilateral pulsating quality, mod to severe intensity, aggravated by routine activity)  has one of the following (Nausea and/or Vomiting, photophobia or phonophobia)  5 or more attacks  no other reason for HA Migraine with aura Rest in dark room w/ ice; avoid triggers (MSG, chocolate, ripened cheese, fermented foods, alcohol, caffeine, sleep changes, stress, menses, skipping meals, odors, bright light, change in weather) Abortive tx: triptans, NSAIDs, antiemetics Prophylactic tx: beta-blockers, TCAs, anticonvulsants (avoid if hx of kidney stones) Contraindications for triptans: ischemic heart disease, CVA, TIA, HTN, diabetes, obese, male > 40, HLD; ↑ risk of serotonin syndrome with SSRI or SNRI; Do not start within 2 weeks of MAOI use; Do not combine with ergots ***Avoid use of combined estrogen- NERVOUS SYSTEM come from same gene  2 attacks with aura  Visual, sensory, motor reversible  Develops over 5-20 min; HA w/in 60 min. (Butterbur, feverfew and magnesium) progestin oral contraceptive in migraine with aura due to stroke risk NERVOUS SYSTEM Name Cause Signs/Symptoms Diagnostics Treatments Concerns Vertigo NOT DIAGNOSIS Symptom of vestibular dysfunction, Spinning, swaying, tilting, N&V, postural instability. Single episode or recurrent Peripheral etiology: involves the vestibular system; severe N/V, recurrent < 1 minute Central etiology: involves the brainstem or cerebellum – prolonged nystagmus, impaired gait/mobility, single episode lasting minutes to hours If you can get the patient to focus on an object and get the vertigo to stop, without shifting gaze – you can keep them. Otherwise refer Benign Paroxysmal Positional Vertigo: brief, recurrent, symptoms are reproducible, attributed to calcium debris in semicircular canals; Dix-Hallpike maneuver; change in position precipitates symptoms Tx with antihistamines like meclizine & Dramamine or Benzos like alprazolam or lorazepam and TIME Polyneuropathy Symmetrical, burning, weakness and sensory loss, variable course, rapid progression, lower extremity more common with symptoms distal to the trunk Risk factors: diabetes and alcohol abuse Tremors Parkinson’s – chronic progressive; average age of 70; tremor at rest, pill rolling tremor, bradykinesia and rigidity.  Levodopa first line treatment – refer Essential Tremor – most common; familial (50% autosomal dominant trait); more common with aging, bilateral action tremor of hands, forearms, head, voice, chin and lip tremor – tremor in legs in unusual.  Betablocker Multiple Sclerosis Immune mediated, inflammatory, demyelinating disease of CNS Young adult with abnormal limb sensation, visual loss, motor symptoms, diplopia, gait disturbance, acute motor symptoms MRI Refer to neurology Dementia Decline in cognition Insidious; decline in complex attention, executive function, learning memory, perceptual motor, social cognition. Cognitive deficits must be severe enough to interfere with function and independence Normal or abnormal cognition? Dementia, delirium or depression? MMSE: <24 suggestive dementia Medications that impair cognition: analgesic, anticholinergic, psychotropic, sedative- hypnotics CBC, CMP, B12, folate, TSH, UA, RPR, HIV, CT and/or MRI Screen for depression Declining cognitive function need safety assessments:  Driving  Financial capacity  Wandering  Living alone Caregiver burnout, polypharmacy, family conflict over decision making, risk of injury, elder abuse Most common cause is Alzheimer disease followed by Lewy bodies. Delirium Prescription meds, substance abuse, drug- drug interaction, abrupt drug withdrawal, preexisting medical condition, infections, electrolyte Reversible, temporary process. Duration is usually brief (hours to days). Pt may be excitable, irritable, combative, short attention span, memory loss and disorientation. Remove and or treat illness, infection or metabolic derangement Sundowning – occurs in delirium and dementia. Starting at dusk, the patient becomes agitated, confused and combative and symptoms resolve in the morning. Avoid dark, quiet spaces. imbalance, heart failure, renal failure Use radio, do not move furniture or décor. ANEMIA  Complex of signs and symptoms o ↓RBC, ↓ hemoglobin, ↓hematocrit  Acute blood loss (uncommon in primary care)  Chronic blood loss (common in primary care) o Erosive gastritis, menorrhagia, GI malignancy o Leads to IDA  Reduced RBC production (sick bone marrow) o Vitamin B12, folic acid, iron deficiency, anemia of chronic disease, bone marrow suppression, reduced erythropoietin production (chronic renal failure) – select medications (PPI, metformin)  Premature destruction (uncommon) o Shortened RBC lifespan (90-120 days normal) o Sickle cell anemia, thalassemia, hemolytic G6PD deficiency  In a person with normal bone marrow production, supplementing the deficient substance will cause H/H to increase in 1-2 weeks and normalize within 4- 8 weeks HEMOGRAM EVALUATION IN ANEMIA  What are hematocrit, hemoglobin, and RBC values? o Normally hemoglobin to hematocrit ratio 1:3  10 = 30%  12 = 36%  15 = 45% o Severe dehydration causes elevated hematocrit o Testosterone = more RBC (hence male have higher hematocrit values)  What is the RBC size? o RBC size is same size during life o MCV (cytic = size)  Microcytic <80  Normocytic 80-96  Macrocytic > 96 o In evolving microcytic anemia  As MCV ↓ RDW ↑ o In evolving macrocytic anemia  As MCV ↑ RDW ↑  What is the RBC hemoglobin content? o MCH or MCHC (chromic = color) o Normochromic: 31-37 o Hypochromic: <31  What is RDW (RBC distribution width)? o Index of variation in RBC size o Abnormal = > 0.15 proportion (15%)  New cells differ in size  Early indicator of evolving microcytic or macrocytic anemia  What is reticulocyte percentage? o Body attempts to correct anemia o Normal 1-2% o Response to anemia is >2% o Reticulocytopenia - Low means body cannot fix the anemia LABS Serum iron – measure of iron in circulation Serum ferritin – < 15 (iron in storage) Chronic smokers, COPD, high altitudes Hct more than 48% women and 52% men Hbg more than 16.5 women and 18.5 men Reticulocyte count – indicates ability of bone marrow to produce RBCs TIBC – total iron binding capacity TIBC is ↑ when iron ↓ TIBC is ↓ when iron ↑ High Altitude stress – low barometric pressure causes reduction in arterial PO2 MICROCYTIC ANEMIAS Low MCV; Low MCHC Thalassemia Iron-deficiency Lead poisoning Anemia of chronic disease MACROCYTIC ANEMIAS Low H/H; High MCV B12 deficiency Folic acid deficiency Impaired liver Thyroid hypo-function Reticulocytosis Drug induced Macrocytosis without anemia (ETOH >5 drinks in men > 3 drinks in women; carbamazepine, valproic acid, phenytoin; malabsorption, zidovudine (reversible, but meds outweigh the risks – DC ETOH – heavy alcohol intake has swollen cells) Hgb, Hct, RBC, MCHC, RDW) normal with ↑ MCV HEMATOLOGICAL SYSTEM HOW BLOOD CELLS MADE  RBC, WBC, Platelets come from Stem cell  Infection o Bacterial or allergic  Neutrophil - infection  Eosinophil – allergic reaction  Basophil - anaphylaxis o Viral  Lymphocyte o Tissue damage  Monocyte  When poly/lymph close = viral  When poly/lymph far = bacterial  Pt should look like labs. nystatin Anemia of Chronic Disease ↓ renal EPO productio n Most common type of anemia in elderly followed by IDA and then pernicious anemia. Hgb ↓ 12 women; ↓ 13 men Normocytic, normochromic anemia, reticulocytopenia Check serum ferritin, TIBC, vitamin B12 and folate Treatment aimed at control of underlying disease or diagnosing the occult disease or illness Name Cause Signs/Symptoms Diagnostics Treatments Concerns Thalassemia Minor Genetic producing abnormal Hgb Majority asymptomatic. Discovered due to abnormal CBC results revealing microcytic/hypochromic RBCs ***Cooley’s anemia is Beta Thalassemia Major – transfusion dependent anemia (found early in infancy) ↓ Hgb ↓ Hct ↑ RBC ↓ MCV ↓ MCHC Normal RDW Normal to ↑ ferritin and iron Normal TIBC Diagnostic test: Hemoglobin electrophoresis Beta-thalassemia (abnormal) IDA, normal (Blood smear: microcytosis, anisocytosis, poikilocytosis) Do not treat thalassemia Genetic counseling prior to pregnancy If someone comes up positive screen the family At risk ethnic groups Alpha thalassemia: Asian, African ancestry (AAA) Beta thalassemia: African, Mediterranean, Middle Eastern (more common in US) Aplastic Anemia Destruction of stem cells inside bone marrow (radiation, drug, viral infection) Bone marrow production slows or stops Fatigue, weakness, pale color, tachycardia and systolic murmur; neutropenia, thrombocytopenia CBC w/ diff Platelet county Bone marrow biopsy (gold standard) Refer to hematologist Pancytopenia (leukopenia, anemia, thrombocytopenia ) Pernicious anemia Autoimmune causing destruction of parietal cells; gastrectomy, vegans, alcoholics, bowel disease Gradual onset of paresthesia on feet/hands, pallor, glossitis; numbness/tingling extremities, neuropathy, diff fine motor skills B12/folate levels; B12 levels may be normal in 5% of patients with B12 deficiency Antiparietal and anti-intrinsic factor (IF) antibody test + 24h urine for methylmalonic acid, homocysteine level elevated, peripheral blood smear (macrocytosis) Dietary deficiency may > 5 years to occur B12 sources: foods of animal origin (meat, poultry, eggs, milk, cheese) B12 via injections or nasal spray (1000mcg per week for 4 weeks then monthly for a lifetime) Oral 1000-2000mg daily Multivitamin with iron since IDA commonly coexist Vitamin B12 deficiency (pernicious anemia, gastric disease, infections, antacids and metformin) Nerve damage from chronic B12 deficiency ↑ incidence in older women 2-3x ↑ gastric cancer **All dementia or patients with neuropathy need B12 levels checked Folic acid deficien cy Inadequate dietary intake causing damage to DNA or RBCs Anemia, tired, fatigue, pallor, reddened sore tongue, glossitis, unexplained weakness, possible tachycardia, palpitations, angina or heart failure. Macrocytic normochromic, peripheral smear – macroovalocytes folate levels <4 Body’s supply last 2-3 months Lifestyle changes (dietary – leafy green vegetables, grains, beef, liver) PO folic acid 1-5 mg/day Pregnancy 400 mcg daily Elderly, infants, alcoholics, overcooked vegetables, low citrus intake, malabsorption (gluten). Drugs that interfere: Phenytoin (Dilantin), sulfa, metformin, methotrexate, zidovudine Sickle cell anemia Genetic hemolytic anemia; variations in RBC – sickle shaped and insufficient of oxygen carrying capacity Most asymptomatic; extreme anemia, frequent sickling episodes w/ pain, ischemic necrosis of bones or skin, renal/liver dysfunction, priapism, hemolytic episodes, hyposplenism, frequent infections – highly susceptible to infection. If fever give prophylactic PCN up to age 5 CBC Sickledex – screening Electrophoresis – gold standard Mean Hgb 8.0 RBC live 17 vs. 120 days Refer to hematologist, Sickle cell disease is part of newborn screening; autosomal recessive (if each parent trait – one of four will have disease) prenatal screening available as early as 8-10 weeks via chorionic villus sampling or amniocentesis 1 out of 500 African Americans in US have sickle cell anemia ↑ risk Strep pneumo, H. influenzae due to hyposplenia HEMATOLOGICAL SYSTEM Characteristic RA OA Primary joint affected Hands; metacarpo- phalangeal Weight bearing, carpometa- carpal, DIP Heberden’s Nodes Absent Usually present Joint Description Soft, warm, tender Bony and hard Labs: RF, CCP, ESR, CRP Positive negative RANDOM PEARLES  Pathological fracture – may be related to osteosarcoma or osteoporosis  Stress fracture – overuse injury of bone o Take 6 weeks to heal  DeQuervain’s Tenosynovitis – dorsal thumb pain o Use fingers a lot  Contusion – bone injured but didn’t break  Strain – injury to muscle  Sprain – injury to ligament  Cauda Equida syndrome – compression of spinal cord  Vitamin D is recommended for all adults > 50  Risk factors for ankle sprain include: o Poor conditioning o Inappropriate footwear o Lack of warm-up period prior to exercising FIBROMYALGIA Mechanism unknown 4-7x more common in women More common in patients with autoimmune disease Widespread body aches, fatigue and cognitive changes Diagnosis involves identifying multiple tender points throughout body 11/18 o Apply enough pressure so that nailbeds blanch Physical activity aimed at ↑ flexibility Trigger point injection may be helpful Acetaminophen, NSAIDs, Trazodone, antidepressants, antiepileptics SPORTS PARTICIPATION EVALUATION  Cardiovascular evaluation is an important component of sports physical o < 35 yrs. are mostly caused by cardiac malformations  Hypertrophic cardiomyopathy o >35 yrs. are atherosclerotic CAD o Cardiovascular hx should include:  Prior occurrence chest pain/discomfort, syncope  SOB or fatigue with exercise  Hx of heart murmur or elevated BP  Family hx premature death, cardiovascular disease <50 yrs. of age o Cardiovascular physical examination  Precordial auscultation supine and standing  Assessment of femoral artery pulses to rule out coarctation of aorta  BP sitting and standing  HTN – avoid beta-adrenergic antagonist because of ability to blunt normal increase in heart rate; avoid diuretic if possible due to ↑ risk of dehydration  Physiological murmur is ok – eval by cardiology  Aortic Stenosis – play varies with degree  Mitral Stenosis – play varies with degree  Mitral regurgitation – mitral valve incompetency – regurgitation from left ventricle to left atria; commonly caused by rheumatic fever, endocarditis, calcific annulus, rheumatic heart disease, some mitral stenosis is usually present – play varies with degree of mitral regurgitation and ventricular chamber enlargement  Mitral Valve Prolapse – most common valvular heart problem – common in pectus excavatum; ok to play if absence of symptoms of activity intolerance  Hypertrophic cardiomyopathy – disease of cardiac muscle – can lead to sudden cardiac death; mid-systolic murmur that gets louder with standing  Those with ICD should be aware of risk  Septal/Atrial Defect – if repaired with little residual dysfunction, full sports is allowed; if no repair then degree should be assessed on individual basis; easily fatigued is a sign of atrial septal defect; child presentation can range from entirely well to heart failure  A still murmur has a buzzing quality  S2 split is occasionally found in uncorrected atrial septal defect  Sinus Arrythmia should be encouraged to play  Down syndrome needs cervical spine x-ray prior to sports (AAI) MUSCULOSKELETAL TYPES OF SCANS  X-rays – bone fractures/damage, OA, metal, dense objects  MRI – gold standard for injuries to cartilage, meniscus, tendons and ligaments o No metal, pacemakers, aneurysm clips  CT – costs less than MRI, views structures likes masses, trauma, fractures, bleeding. Forms 3- D picture Hook Test – Biceps Tear – MRI ORTHO TERMINOLOGY  Abduction (varus) – movement away from body  Adduction (valgum) – movement toward the body Name Cause Signs/Symptoms Diagnostics Treatments Concerns Navicular Fracture (Scaphoid) Fall with outstretche d hand Pain on palpation of “snuffbox” area, pain on axial loading of thumb; pain worse when gripping or squeezing x-ray initially may be normal; repeat in 2weeks Splint wrist (thumb spica splint) refer to hand surgeon Avascular necrosis and nonunion Colles Fracture Fracture of distal radius from fall “dinner fork” fracture. Most common type fracture Hip Fracture Falls Sudden onset on-sided hip pain. If mild may be able to bear weight. If displaced, inability to walk or bear weight on affected hip. Severe hip pain with external rotation of the hip/let and leg shortening More common in elderly 1-year mortality rate from 12-37% Pelvic Fracture High-energy trauma (MVC) Depends on degree of injury and structures i.e.: nerves, blood vessels, organs. Ecchymosis and swelling in lower abdomen, hip, groin, scrotum. Bladder/fecal incontinence, vaginal/rectal bleeding, hematuria, numbness. May cause internal hemorrhage - life- threatening Cauda Equina Syndrome Acute onset saddle anesthesia, bladder/fecal incontinence. Bilateral leg numbness and weakness, pressure on sacral nerve root causing inflammatory/ischemic changes to nerves. Surgical emergency Refer to ED Needs spinal decompression Medial Tibial Stress Syndrome or Fracture (Shin Splints) Overuse resulting in microtears and inflammation of muscles, bones and tendons Recurrent shin pain in one or both legs that becomes more severe over time. Pain along inner border of tibia and occurs during and after exercise. Mild swelling and focal area of tenderness painful on palpation may suggest fx X-ray will not show stress fx. Recommend bone scan or MRI RICE – Stop activity for several weeks Cold packs during acute exacerbation for 20 minutes several times a day Use cushioned soles Stretch before exercise and start at lower intensity More common in runners and those with flat feet. Female athlete triad: amenorrhea, eating disorder, osteoporosis Plantar Fasciitis Microtears of plantar fascia Plantar foot pain, unilateral or bilateral, worsened by walking or weight bearing. worse in morning or with prolonged walking x-ray to r/o fractures, spurs NSAIDs, Naproxen, diclofenac gel, orthotics, stretching. Refer to podiatry ↑ risk with obesity, diabetes, aerobic exercise, flat feet, prolonged standing Morton’s Neuroma Inflammation of digital nerve of the foot between 3rd and 4th metatarsal (nerve tumor) Weeks of plantar foot pain worsened by walking (esp. high heels or tight narrow shoes); pain has numbness/burning located between 3rd/4th metatarsals on forefoot. “pebble-like” nodule Mulder test (MTP squeeze) - grasp 1st/5th metatarsal and squeeze. Positive if click along with patient report of pain Avoid narrow/high shoes, use forefoot pad, well-padded shoes. Refer to podiatry ↑ risk with high heeled shoes, tight shoes, obesity, dancers and runners Degenerative Joint Disease (Osteoarthritis) Damage of articular covered Gradual onset: early-morning joint stiffness w/ inactivity. Short duration (<15 min). pain w/ overuse, swelling, tender to palpation. DIP common, May be unilateral – absence of systemic symptoms; pain awakens at night Heberden’s nodes (nodules on x-ray- joint effusion, osteophytes and joint space narrowing r/o osteoporosis w/ bone- density testing Goals: Relieve pain, preserve joint mobility and function, minimalize disability and protect joint PT/exercise – weight bearing; weight loss if appropriate, Acetaminophen then Large weight bearing joints (hips and knees) and hands are most commonly affected progression and resolution of disease. Fever, fatigue, anorexia and arthralgias, rash, lesions, color change, nodule formation under skin, blurred vision, eye pain, severe redness and sensitivity to light ACE Glycoprotein KL-6; Hypercalcemia; Hypercalciuria Chest x- ray; CT chest; PFT; biopsy Corticosteroids orally, cream, inhaled Plaquenil, DMARDs Lung transplant production of Adults 20-40; women and Sarcoidosis noncaseating African Americans more granulomas common predominately lungs, lymph nodes, eyes, skin Reactive arthritis (formerly Reiter Syndrome) Acute nonpurulen t arthritis 2+, with at least one musculoskeletal: asymmetrical oligoarthrtitis, predominately lower extremity, sausage shaped finger (dactylitis); toe or heel pain; cervicitis, prostatitis, acute diarrhea within one month, conjunctivitis or uveitis, genital ulceration, urethritis; joint pain knee/ankle/feet With diarrhea affects genders equally; with urethritis (male dominance) with HLA-B27 positive; culture of joints negative NSAIDs, systemic corticosteroids, tumor necrosis factor blocker (etanercept or infliximab); Urethritis treated with doxycycline x 7 days or azithromycin single dose Often seen days to weeks after diarrhea caused by Shigella, Salmonella or Campylobacter or Chlamydia TCA Not first line for depression Postherpetic neuralgia, stress incontinence Avoid if ↑ risk for suicide Overdose = fatal cardiac, neurological effects, SE = Anticholinergic, hypotension, conduction arrhythmia, glaucoma, BPH, confusion Migraine prophylaxis Imipramine, amitriptyline and nortriptyline Avoid in CV disease, elderly (BEERS) SDRI Bupropion (Wellbutrin) avoid seizures/bulimia Usually used as add-on to SSRI Best effect on improving mood with insufficient response with SSRI ***20% chance sexual adverse effects PSYCHOSOCIAL MENTAL HEALTH MAOI  Food and drug interactions  Phenelzine (Nardil) and tranylcypromine (Parnate)  Do not combine with SSRI, TCA, triptans  Elevates BP and risk of stroke when used with fermented foods such as beer, wine, cheese ANTIDEPRESSANT DISCONTINUATION  SSRI, SNRI, TCA > 6 months then discontinued. Typically, last < 7 days. Taper over 6 weeks to reduce symptoms. Bothersome, not life threatening.  F – Flu-like symptoms  I – Insomnia  N – Nausea  I – Imbalance (dizziness, difficult coordination)  S – Sensory disturbance  H – Hyperarousal (anxiety/agitation)  H – Headache ANXIOLYTICS  Benzodiazepines (use long acting to avoid abuse) o ↓ dose by 25% each week when dc  Buspirone (BuSpar) -low abuse potential o Potentially helpful if taken 3x daily for 6 weeks or longer (useless as prn & for sleep) QUESTIONS TO ASK PRIOR TO RX  What are the most bothersome symptoms?  What meds will help with these symptoms? COMMON SNRIS  Venlafaxine (Effexor) ↑ BP  Duloxetine (Cymbalta) neuropathy (avoid in alcohol users)  Desvenlafaxine (Prestiq) Best effect on lifting and smoothing mood plus increase focus ***40% chance sexual adverse effects TREATMENT GOALS ANXIETY/DEPRESSION  Remission of symptoms for > 4-5 months aimed at elimination and restorative health o Most often achieved with psychologic, social services and medications o Slowly taper off meds DEPRESSION SYMPTOMS  S – Sleep – insomnia or hypersomnia  I – Interest– less interest in activities, irritability (anhedonia)  G – Guilt – worthlessness or inappropriate guilt  E – Energy – fatigues  C – Concentration – diminished  A – Appetite – weight change, loss or gain  P – Psychomotor – agitation or retardation  S – Suicide – recurrent/obsessive thoughts  M – Mood – depressed mood, tearful Major depression: > 5 symptoms Minor depression: 2-5 symptoms r/o hypothyroidism, anemia, autoimmune, B12 def. S/S in teens: failing grades, acting out, avoiding socialization, moodiness. ANXIETY SYMPTOMS  W – Worry  A– Anxiety  T – Tension in muscles  C – Concentration difficulty  H – Hyperarousal or irritability  E – Energy loss  R – Restless  S – Sleep disturbance  > 3 of the following occurring on most days > 6 months COMMON SSRIS (listed from most to least energizing) First line tx for major depression, OCD, anxiety and premenstrual disorder  Fluoxetine (Prozac) longest ½ life (interacts with coumadin)  Sertraline (Zoloft)  Citalopram (Celexa) few drug interactions, QT prolongation (max dose 20 elderly)  Escitalopram (Lexapro)  Paroxetine (Paxil) shortest ½ life (lots drug interaction and sedating – Erectile Dysfunction and anticholinergic effects) Best effect on lifting and smoothing mood ***Can induce mania w/ bipolar. Do NOT IN C R EA SED R ISK O F SU IC ID A L TH IN K IN G IN TH O SE LESS TH A N 2 4 Y EA R S PSYCHOSOCIAL MENTAL HEALTH SEROTONIN RECEPTOR SITES  5-HT1A – antidepressant  5-HT1C, 5-HT2C – cerebral spinal fluid production  5-HT1D – antimigraine effect (anti=defense)  5-HT2 – agitation, anxiety, panic (2 stressed)  5-HT3 – nausea, diarrhea (3 GI N/V/D) MOOD  Monoamine System o Serotonin (5-HT)  Well being  Calm  ↓ impulsivity  ↓ sex drive  ↓ aggression  ↑ appetite o Dopamine  Make you worry  ↑ vigilance  ↑ motivation o Norepinephrine  Enhance concentration  Enhance ambition  Enhance productivity ANTIPYSCHOTICS SIDE EFFECTS  Pill rolling, shuffling gait, bradykinesia  Extrapyramidal symptoms: o Akinesia - inability to initiate movement o Akathisia - strong inner feeling to move, unable to stay still o Bradykinesia - slowness in movement when initiating activities that require successive steps such as buttoning a shirt o Tardive dyskinesia - involuntary movements of lips (smacking), tongue, face, trunk and extremities  Increased risk of obesity, Type 2 DM, hyperlipidemia, metabolic syndrome ALTERNATIVE MEDS FOR DEPRESSION  St. John’s wort o Interacts with SSRI, TCA, MAOI o ↓ Digoxin effectiveness o ↓ effectiveness of birth control  5-HTP, L-tryptophan o Interacts with SSRI, MAOI, dextromethorphan, Triptans  Omega-3 fatty acids o No major drug interactions o High doses may ↑ risk of bleeding o Stop 1 week before surgery  Folate and vitamin B6  Exercise, yoga, massage, guided imagery, acupuncture, light therapy  Kava-Kava and valerian root are both used for anxiety and insomnia. Do not mix with benzos, hypnotics or any CNS depressants SUICIDE  Males represent nearly 80% of all completed suicides.  Females attempt suicide 2-3 times more often.  Highest rate of completed suicide is found in elderly males (75+).  Inquiring about suicidal ideation DOES NOT lead to suicide.  Risk Factors: o Older people who have lost a spouse o Plan involving gun or lethal weapon o Hx of attempted suicide o Mental illness: bipolar, depression o Hx of sexual, emotional or physical abuse o Terminal illness, chronic illness, pain o Significant loss o Bipolar is higher risk during depressive episode  3 important questions: o Are you thinking of hurting yourself? o If yes, do you have a plan? o If yes, do you have the means?  Imminent risk o Immediate psychiatric referral, inpatient hospitalization  Elevated risk but not imminent o Aggressive treatment ALCOHOL SCREENING  C: Do you feel the need to cut down?  A: Are you annoyed when your friends/spouse comment about your drinking?  G: Do you feel guilty about your drinking?  E: Do you need to drink early in the morning?  Positive response to 2/4 is highly suggestive of alcohol abuse  Anyone feeling compelled to drink no matter what the consequences is addicted PSYCHOSOCIAL MENTAL HEALTH Name Caus e Signs/Symptoms Diagnostics Treatment s Concerns Insomnia Circadian rhythm disorders, psychic issues, mental illness, OSA, RLS, environment al factors, certain medications, idiopathic. 7-8 hours of sleep is the ideal amount. Difficulty falling asleep or waking up during night too early and unable to go back to sleep; daytime drowsiness, fatigue, headache, irritability, difficulty concentrating Primary – not caused by disease or environment Secondary – caused by disease or environment Short term – less than 3 months, pain, stress, grief Chronic – at least 3 months, occurs at least 3 nights per week Sleep hygiene (regular time, avoid caffeine, Refer to sleep lab (polysomnography is gold standard for sleep apnea) Antihistamines (careful w/ Benadryl in elderly) Benzo hypnotics: Short acting: Alprazolam, Triazolam, Midazolam Intermediate acting: Lorazepam, temazepam Long acting: Diazepam, clonazepam, chlordiazepoxide (Halcion and temazepam are more sedating) Non-Benzo hypnotics: adverse effects include agitation, hallucinations, nightmares, suicidal ideations. Awakening and cannot recall event. Zolpidem (Ambien) – sleep onset or inability to stay asleep Eszopiclone (Lunesta) – sleep onset or inability to stay asleep Ramelteon (Rozerem) – sleep onset insomnia (melatonin agonist) Complementary: Kava-Kava, Valerian root (do not give to children or lactating/pregnant & do not mix with benzos or hypnotics), Melatonin, Chamomile tea, meditation, yoga, Tai-Chi, acupuncture, regular exercise (avoid 4 hours prior to bedtime) Risk factors: depression, anxiety, GERD, female, illicit drug use, musculoskeletal illness, pain, chronic health problems, shift work, alcohol, caffeine and nicotine. (certain medications – SSRI, cardiac, BP, allergy and steroids can cause insomnia) Schizophrenia Delusions and paranoia (disorganized speech and behavior). Hallucinations are common (usually auditory) with loss of ego boundaries; flat and restricted affect with poor social skills, ability to plan and organize day-to-day activities (executive skills) are poor Refer to psychiatrist Use of typical antipsychotics can increase sudden death Antipsychotics can prolong QT intervals (EKG needed) and can cause torsade de pointes – clozapine, thioridazine, ziprasidone, haloperidol and others Onset is usually 16-30’s Anorexia Nervosa Irrational preoccupation with intense fear of gaining weight with distorted perception of Secretive, perfectionistic and self- absorbed. Marked weight loss (BMI <18.5), lanugo on face, back, shoulders, amenorrhea for 3 mon or longer, abdominal distention with hepatomegaly, cheilosis, oral and gum disease, coarse dry skin, hypotension with bradycardia and hypothermia DSM V:  Inability or refusal to maintain body weight at or above minimum normal weight for age and height  Intense fear of SSRI 1st line Wellbutrin contraindicated– increases seizure threshold Chvostek’s sign – contract of facial muscles when facial nerve is tapped briskly Onset usually during adolescence Osteopenia/osteoporosis due to prolonged estrogen depletion from amenorrhea and low calcium intake, peripheral edema due to body shape and weight If purging - loss of dental enamel. Engage in severe food restriction or binge eating and purging. Some use laxatives, vomiting and excessive daily exercise. Onset teens to early 20’s gaining weight and becoming fat despite low body weight  Disturbance in perception of body weight and shape – associated with hypocalcemia which could further develop to tetany low albumin from low protein intake, cardiac complications – arrhythmia, cardiomyopat hy, hypokalemia PSYCHOSOCIAL MENTAL HEALTH Name Caus e Signs/Symptoms Diagnostics Treatment s Concerns Bulimia Nervosa Secretive disease. Problems with erosion of lingual surface of upper teeth due to excessive exposure to gastric contents during vomiting. Hypokalemia caused by laxative and diuretic use is common. Pt is typically of average to slightly above average weight. DSM V:  Eating excessing amount of food in 2 hours  Person feels lack of control over eating  Recurrent compensatory behavior to prevent excessive weight gain such as vomiting, excessive exercise, laxative or diuretic abuse or fasting  Occurs once per week for at least 3 months Cognitive behavior and pharmacological therapy. SSRIs – but NOT Wellbutrin Binge eating is lack of control over amount and type of food 2+ times per week for 6 months. Accompanied by distress, self-anger, shame and frustration because of purging. Pt is usually obese. PTSD Combat/war, sexual assault, MI, stroke, ICU stay Flashbacks, nightmares, intrusive thoughts, avoidance of reminders of trauma, agoraphobia, sleep disturbance and hypervigilance, feelings of detachment Assessment tools such as PTSD checklist First line treatment is SSRI such as paroxetine and sertraline. Therapeutic trial of 6-8 weeks to determine effectiveness. Mirtazapine for sleep. Cognitive behavioral therapy, Eye movement desensitization and reprocessing (EMDR). Comorbidities such as depression, anxiety, antisocial disorder and substance abuse is higher Anxiety Situational, Phobias, OCD, Generalized Panic disorder occurs more in women than men with agoraphobia GAD – excessive worry occurs on more days than not for 6 months Benzos – limited duration, addiction concern behavioral treatment and SSRI to kick in and then wean off Benzo GAD – SSRI, SNRI, buspirone Panic disorder – SSRI, SNRI, TCA, BB, MAOI OCD – SSRI, SNRI, TCA Herbs: Kava-kava, valerian root and passion flower Munchaus en Syndrome Factitious disorder imposed on self. Patient falsifies symptoms and or injures self-seeking medical treatment. Munchausen syndrome by proxy (makes the child sick) RENAL URINALYSIS Epithelial Cells – large amounts indicate contamination; a few are normal Leukocytes – normal WBCs in urine <10 Leukocyte esterase Pyuria (presence of leukocytes) in males is always abnormal Urine for Culture and Sensitivity o >100,000 Red Blood Cells o Protein o o <5 is normal  Indicates kidney damage Urine dipstick detects albumin not microalbumin (Bence-Jones proteins)   Nitrites o Cast o o o Indicative of infection Hyaline cast are normal WBC cast may be seen with infection RBC cast and proteinuria are diagnostic of glomerulonephritis KIDNEYS  Prerenal azotemia – most common cause of acute renal failure, kidneys are hypoperfused – which leads to acute tubular necrosis. Caused by ↓ circulating volume such as dehydration and acute blood loss; ↓ CO such as heart failure; excessive sequestering of fluids as in burns  Postrenal azotemia – obstruction to urine flow and is uncommon cause of renal failure. Such as glomerulonephritis CHRONIC KIDNEY DISEASE  Common electrolyte disorders include hypernatremia, hypercalcemia and hyperkalemia  Increase in creatinine from 1-2 indicates a 50% loss in renal function  Creatinine clearance usually approximates eGFR  Creatinine is best described as a product related to skeletal muscle metabolism  Common causes include DM, recurrent pyelonephritis, polycystic kidney disease  Persistent proteinuria is commonly found in early development of CKD  ACEI can limit the progression of some renal disease by reducing efferent arteriolar resistance  Objective findings in glomerulonephritis include edema, RBC cast and proteinuria  Anemia: Normocytic, normochromic anemia with low retic count  Erythropoiesis is recommended with CKD and Hgb < 10  Dialysis and transplant discussion at Stage 4 CKD  Some meds that affect kidneys – Allopurinol, antibiotics, digoxin, lithium, gabapentin, H2 blockers, anti- arrythmias KIDNEY FUNCTION  Serum Creatinine – when renal function ↓ creatinine ↑ Creatinine affected by age (less sensitive in elderly), gender (higher in males), ethnicity (high with African background), muscle mass Male 0.7 to 1.3 Female 0.6 to 1.1  Estimated Glomerular Filtration Rate (eGFR) eGFR “estimated value” – more damaged the kidneys, the lower the eGFR. Best if patient does not eat meat 12 hours before test and is less reliable with drastic changes in muscle mass, pregnancy and acute renal failure normal eGFR > 90 Stage 2 eGFR 60-89 Stage 3a eGFR 45-59 Stage 3b eGFR 30- 44 Stage 4 eGFR 15- 30 Stage 5 eGFR < 15  Blood Urea Nitrogen – elevation may be caused by acute renal failure, high-protein diet, hemolysis, CHF or drugs (waste product of protein from foods eaten, dehydration will also elevate BUN)  BUN-to-Creatinine Ratio – evaluate dehydration, hypovolemia, acute renal failure KIDNEYS  Body’s regulator of fluids  Water is reabsorbed by antidiuretic hormone and aldosterone  Excrete water-soluble waste (creatinine, urea, uric acid)  Produce erythropoietin (stimulates bone marrow to produce more RBC), renin, bradykinin, prostaglandins and calcitriol/vitamin D3  Average UO is 1500 mL  Oliguria < 400 mL day  Right kidney sits lower than left due to liver displacement RENAL Name Cause Signs/Symptoms Diagnostics Treatments Concerns Pyelonephritis E. Coli, Klebsiella spp. Proteus mirabilis High fever, chills, dysuria, frequency, and unilateral flank pain (described as deep ache) N/V May have had recent UTI CVA tenderness UA – large leukocytes, hematuria, WBC cast and proteinuria Urine C&S Uncomplicated may treat as outpatient Cipro BID x 7 days or Levaquin daily Rocephin 1 gram + Augmentin BID x 14 days Acute renal failure Decreased blood flow to kidneys; damage to kidneys; urine blockage in kidney Abrupt onset of oliguria, edema and weight gain (fluid retention). Complains of lethargy, nausea and loss of appetite. Rapid ↓ in renal function ↑ creatinine ↓ GFR Hydrate Bladder cancer Long term use with pioglitazone Painless hematuria (microscopic or gross). May appear at the end of voiding. Dysuria, frequency, nocturia (not related to UTI). Advanced disease may complain of lower abdominal or pelvic pain, perineal pain, low-back pain or bone pain. UA – microscopic hematuria is the primary finding in 20% of individuals with bladder CA Urine C&S Urine for cytology Preferred therapy for nonmuscular- invasive bladder cancer without evidence of metastasis is transurethral resection with intravesical chemotherapy. Despite successful initial therapy, local recurrence is common Risk factors: Elderly > 50; male (73 years), smoker, occupational exposure to textile dyes and heavy metals Hematuria Cancer, infection, renal calculi, coagulopathy, glomerular disease, hydronephrosis, polycystic kidneys, trauma, medications, BPH, exercise induced Gross hematuria if urine is pink, red, brown or blood clots are present. UA If infection: Urine C&S If malignancy: Urine for cytology Urinary Tract Infection (Cystitis) E. Coli, Klebsiella spp. Staph. Saprophyticus, Proteus mirabilis Frequency, burning, urgency, dysuria, hematuria, foul-smelling urine, nocturia, lower abd. /back pain – NO FEVER Risk factors: female, pregnancy, hx of UTI, DM, failure to void after sex, spermicide use, low fluid intake, poor hygiene, catheterization UA moderate to large leukocytes, +/- nitrites, few RBC (inflammation) C&S > 100,000 UTI never normal in male – R/O other causes. 3 or more UTI in 1 year in females – R/O other causes ***nitrates are normal in urine, nitrites can indicate infection Uncomplicated: Bactrim DS BID x 3 days; (sulfa allergic/resistance) Nitrofurantoin (Macrobid) 100mg BID x 5 days Ciprofloxacin (Cipro) 250mg PO BID – no fluoroquinolones in pregnancy or <18yo Pyridium – leaves it orange (avoid in liver disease) Complicated: Keflex, Cipro 500mg BID or Levaquin 750 daily for 7-10 days ***Nitrofurantoin contraindicated with renal insufficiency For men do prostate exam Only treat pregnant women with asymptomatic bacteriuria UTI in pregnant women and children <3 are more likely to progress to pyelonephritis Renal and bladder sono for UTI infants Nephrolithia sis (Urolithiasis ) Majority made of calcium oxalate; Struvite stones are found in those with hx of kidney infection Severe colicky pain that comes in waves. Patient cannot sit still. Pain builds in intensity, lessens and disappears. Associated with N/V. May have gross or microscopic hematuria Stones in upper urethra or renal pelvis may cause flank pain and tenderness whereas stones in lower urethra may cause pain radiating to Toradol injection Increase fluids, strain urine Avoid high-oxalate foods: rhubarb, spinach, beets, chocolate, tea and meats Consider alpha blocker Refer urology Risk factors: family hx of stones, gout, bariatric surgery, high doses vitamin C Meds that cause kidneys stones: HCTZ, topiramate, indinavir RENAL testicle or labia. Both can cause abdominal pain Glomerulonephrit is Inflammation of glomeruli in kidney. Occurs 1-2 weeks s/p bacterial infection Pink or cola colored urine due to hematuria, foamy due to proteinuria, HTN, edema of face, hands, feet and abdomen, possible anemia UA ↑ Protein, RBCs, renal cast ↑ Creatinine and BUN CT scan or kidney sono Confirmatory diagnosis is with Kidney biopsy Acute is often self-limiting. Manage underlying cause and protect kidneys – antihypertensives, antimicrobials, systemic corticosteroids and immune suppressants. Plasmapheresis, dialysis If left untreated, can lead to kidney failure, HTN, electrolyte disorders and nephrotic syndrome Risk factors include infection: bacterial endocarditis, immune disease: Goodpasture’s syndrome, SLE, or vasculitis: polyarteritis or Wegener’s granulomatosis
Docsity logo



Copyright © 2024 Ladybird Srl - Via Leonardo da Vinci 16, 10126, Torino, Italy - VAT 10816460017 - All rights reserved